path cvs - formatted Flashcards

1
Q
  1. 67.APRIL02 A 7-year-old girl presents with a systemic illness after a documented group A streptococcal pharyngitis. Acute rheumatic fever is suspected. Which of the following combination of manifestations IS NOT diagnostic of this condition using the Jones criteria:
  2. Carditis and Sydenham chorea
  3. Migratory polyarthritis of large joints and subcutaneous nodules
  4. Migratory polyarthritis of large joints and Sydenham chorea
  5. Erythema marginatum skin lesions and carditis
  6. Subcutaneous nodules and fever
A
  1. Subcutaneous nodules and fever - F - this only constitutes 1 major and 1 minor criteria

• Criteria apply only to an initial attack
• “High probability” of ARF if evidence of recent Gp A Strep inf (throat cx, streptozyme, or elevated or rising strep Ab titers) and (2 major or 1 major + 2 minor)
Major Minor
Carditis Arthralgia
Polyarthritis Fever
Chorea Elevated ESR
Erythema marginatum Elevated CRP
Subcutaneous nodules Prolonged PR interval

*LW: major criteria "JONES"
J - Joint involvement
O - O looks like a heart, so carditis.
N - Nodules, subcutaneous
E - Erythema marginatum
S - Syndeham corea.

• Rheumatic fever is characterized by a constellation of findings that includes as major manifestations

(1) migratory polyarthritis of the large joints
(2) carditis
(3) subcutaneous nodules
(4) erythema marginatum of the skin
(5) Sydenham chorea– a neurologic disorder with involuntary purposeless, rapid movements

How well did you know this?
1
Not at all
2
3
4
5
Perfectly
2
Q
  1. 65.APRIL02 An obstetric ultrasound examination reveals a fetus with an abnormal pulmonary outflow tract. The pulmonary artery appears to arise from the left ventricle but the AV connections are normal. The aorta appears to arise from the right ventricle. Which of the following statements about this condition is most correct?
  2. Two thirds of cases are associated with a patent foramen ovale or patent ductus arteriosus
  3. A VSD is a rare association (<1%)
  4. Untreated cases often present with pulmonary hypertension in their 2-3 rd decades
  5. Berry aneurysms of the circle of Willis are a common association
  6. Right ventricular hypertrophy only develops when there is associated pulmonic stenosis
A
  1. Two thirds of cases are associated with a patent foramen ovale or patent ductus arteriosus (Those with only a patent foramen ovale or PDA (about 65%), however, have unstable shunts that tend to close and therefore require immediate intervention to create a shunt (such as balloon atrial septostomy) within the first few days of life)
  2. 65.APRIL02 An obstetric ultrasound examination reveals a fetus with an abnormal pulmonary outflow tract. The pulmonary artery appears to arise from the left ventricle but the AV connections are normal. The aorta appears to arise from the right ventricle. Which of the following statements about this condition is most correct?
  3. Two thirds of cases are associated with a patent foramen ovale or patent ductus arteriosus (Those with only a patent foramen ovale or PDA (about 65%), however, have unstable shunts that tend to close and therefore require immediate intervention to create a shunt (such as balloon atrial septostomy) within the first few days of life)
  4. A VSD is a rare association (<1%) (Patients with transposition and a VSD (about 35%) have a stable shunt)
  5. Untreated cases often present with pulmonary hypertension in their 2-3 rd decades (Without surgery, most patients die within the first months of life)
  6. Berry aneurysms of the circle of Willis are a common association
  7. Right ventricular hypertrophy only develops when there is associated pulmonic stenosis (Right ventricular hypertrophy becomes prominent as this chamber functions as the systemic ventricle)

• Transposition implies ventriculoarterial discordance, such that the aorta arises from the right ventricle, and the pulmonary artery emanates from the left ventricle.
• The AV connections are normal (concordant), with right atrium joining right ventricle and left atrium emptying into left ventricle
• The essential embryologic defect in complete transposition is abnormal formation of the truncal and aortopulmonary septa.
• The aorta arises from the right ventricle and lies anterior and to the right of the pulmonary artery;
o in contrast, in the normal heart, the aorta is posterior and to the right.
• The result is separation of the systemic and pulmonary circulations, a condition incompatible with postnatal life, unless a shunt exists for adequate mixing of blood.
o Patients with transposition and a VSD (about 35%) have a stable shunt.
o Those with only a patent foramen ovale or PDA (about 65%), however, have unstable shunts that tend to close and therefore require immediate intervention to create a shunt (such as balloon atrial septostomy) within the first few days of life.
o Right ventricular hypertrophy becomes prominent as this chamber functions as the systemic ventricle.
o Concurrently the left ventricle becomes thin-walled (atrophic) as it supports the low-resistance pulmonary circulation.
• The outlook for infants with transposition of the great vessels depends on the degree of mixing of the blood, the magnitude of the tissue hypoxia, and the ability of the right ventricle to maintain the systemic circulation.
• Without surgery, most patients die within the first months of life.
• Currently, most patients undergo a reparative operation (usually entailing transection and switching of the great arteries as well as the coronary arteries) during the first several weeks of life.

How well did you know this?
1
Not at all
2
3
4
5
Perfectly
3
Q
  1. 100.APRIL02 A 60-year-old man with a 2-year history of myeloma develops cardiac failure associated with a restrictive cardiomyopathy. The most likely cause is:
  2. Drug associated cardiomyopathy secondary to chemotherapy
  3. Amyloidosis with deposition of AA type amyloid 2
  4. Viral cardiomyopathy associated with immune suppression
  5. Amyloidosis with deposition of AL type protein
  6. Simple ischaemic cariomyopathy
A
  1. Amyloidosis with deposition of AL type protein

Restrictive cardiomyopathy is characterised by decreased ventricular compliance, resulting in impaired filling during diastole. It can be idiopathic or associated with distinct diseases that affect the myocardium, principally radiation fibrosis, amyloidosis, sarcoidosis, metastatic tumor, or products of inborn errors of metabolism.
• AL (immunocyte-associated / 1o) - immunoglobulin light chains (lambda)
o cannot distinguish from AA but often involves heart, kidney, GIT, peripheral Ns, skin, tongue,
o also odd sites – eyes, respiratory tract
o Associated diseases: MM & other monoclonal B-cell proliferations
Heart
• More common in AL – amyloidosis, and in (senile systemic amyloidosis?)
• Usually normal but may be firm & enlarged
• Myocardial & subendocardial deposits → conduction abnormalities, restrictive CMO, CCF

Involvement of the cardiovascular system by amyloidosis occurs in four general forms:

  1. The most common presentation of cardiac amyloidosis is that of Restrictive CM. Right-sided findings dominate the clinical presentation; peripheral edema is a prominent finding, whereas paroxysmal nocturnal dyspnea and orthopnea are absent. Amyloid infiltration of the myocardium results in increased stiffness of the myocardium, producing the characteristic diastolic dip and plateau (square root sign) in the ventricular pressure pulse that may simulate constrictive pericarditis. In contrast to the accelerated early left ventricular diastolic filling found in constrictive pericarditis, cardiac amyloidosis is marked by an impaired rate of early diastolic filling.
  2. A second common presentation is congestive heart failure due to systolic dysfunction. Hemodynamic evidence of restriction of ventricular filling may not be prominent in these patients. In some patients amyloid deposition in the atria may be responsible for loss of atrial transport function despite the maintenance of electrical “sinus” rhythm, with the production of congestive heart failure. The course of this form of the disease is often one of relentless progression, usually poorly responsive to treatment. Angina pectoris occurs on occasion despite angiographically normal coronary arteries.
  3. Orthostatic hypotension occurs in about 10 percent of cases. Although most likely due to amyloid infiltration of the autonomic nervous system or of blood vessels, amyloid deposition in the heart and adrenals may contribute to the pathogenesis of this variant. Hypovolemia as a result of the nephrotic syndrome secondary to renal amyloidosis may aggravate the postural hypotension.
  4. An abnormality of cardiac impulse formation and conduction is the fourth and least common mode of presentation and may result in arrhythmias and conduction disturbances. Sudden death, presumably arrhythmic in origin, is relatively common and may be preceded by episodes of syncope
How well did you know this?
1
Not at all
2
3
4
5
Perfectly
4
Q
  1. 71.APRIL02 A 54-year-old man with headache has a normal CT head with intravenous contrast. He leaves your hospital after 15 minutes but returns 1 hour later as an unwitnessed collapse, dead on arrival. Autopsy of the heart reveals a mottled slightly yellow area in the anterior wall of the left ventricle. Histology shows necrotic myocytes with an intense acute inflammatory infiltrate devoid of granulation tissue. This is mostly likely to represent;
  2. Contrast associated myocarditis
  3. A hyper-acute myocardial infarct (1 – 2 hours old)
  4. A myocardial infarct less than 24 hours old
  5. A myocardial infarct of approximately 48 hours old
  6. A myocardial infarct of approximately 7 days old
A
  1. A myocardial infarct of approximately 48 hours old

12-24 hr Dark mottling
Ongoing coagulation necrosis; pyknosis of nuclei; myocyte hypereosinophilia; marginal contraction band necrosis; beginning neutrophilic infiltrate

1-3 days Mottling with yellow-tan infarct center
Coagulation necrosis, with loss of nuclei and striations; interstitial infiltrate of neutrophils

3-7 days Hyperemic border; central yellow-tan softening
Beginning disintegration of dead myofibers, with dying neutrophils; early phagocytosis of dead cells by macrophages at infarct border

How well did you know this?
1
Not at all
2
3
4
5
Perfectly
5
Q
  1. 69.APRIL02 A 27-year-old male presents with hypertension in the upper extremities and , weak pulses in the lower limbs. A chest radiograph shows an abnormal aortic arch contour and inferior rib notching. Which of the following IS A LEAST LIKELY association;
  2. Bicuspid aortic valve
  3. Berry aneurysms of the Circle of Willis
  4. Cor pulmonale
  5. Mitral regurgitation
  6. Atrial septal defect
A
  1. Cor pulmonale

Coarctation Associations¬
• bicuspid aortic valve in 50 % congenital aortic stenosis
• ASD
• VSD
• mitral regurgitation
• Berry aneurysms of the circle of Willis

How well did you know this?
1
Not at all
2
3
4
5
Perfectly
6
Q
  1. 60.APRIL02 A patient has a cardiac scan two days after confirmed myocardial infarct. This shows that an area of viable myocardium in an area that is non-contractile on echocardiography. This is best explained by:
  2. Hibernating myocardium
  3. Stunned myocardium post infarct
  4. Luxury perfusion post infarct
  5. Hyperdynamic myocardium adjacent to the infarct
  6. Troponin cross reactivity
A
  1. Stunned myocardium post infarct ACUTE
  • Hibernating myocardium is viable myocardium with depressed contractile dysfunction resulting from chronic ischemia (months to years) insufficient to cause tissue necrosis. Contractility recovers quickly once coronary flow is restored.
  • Stunned myocardium exhibits sustained (hours to days) reduced regional contractility after coronary reperfusion from an acute ischemic insult. However, it will respond to inotropic stimulation, and this behavior of the myocardium can be exploited to differentiate between stunned and infarcted myocardium.
  • Low-dose dobutamine stress echocardiography (0–20 µg/kg per minute) has been used to test for viability in the form of transient recovery of function.
  • Assessment for reversible dysfunction is important for prognosis and to identify high-risk patients, who may benefit most from aggressive revascularization therapy.
How well did you know this?
1
Not at all
2
3
4
5
Perfectly
7
Q
  1. 61.APRIL02 A 42-year-old man is referred for a nuclear heart scan and cardiac enzyme assay. He had 4 hours of “severe reflux” one week earlier. The enzyme diagnosis of a week old myocardial infarct best be supported by finding elevations of which of the following;
  2. CK MB (Creatinine kinease MB)
  3. LDH (Lactate dehydrogenase)
  4. LDH and AST (lactate dehydrogenase and aspartamine transaminase)
  5. AST and CK (aspartamine transaminase and creatinine kinease)
  6. LDH and TnT (Tn) (Lactate dehydrogenase and Troponin T)
A
  1. LDH and TnT (Tn) (Lactate dehydrogenase and Troponin T)
How well did you know this?
1
Not at all
2
3
4
5
Perfectly
8
Q

.21 Man with right sided heart failure LEAST LIKELY finding is ?

  1. Leibmann Sachs endocarditis (SLE)
  2. Arrythmogenic right ventricular cardiomyopathy
  3. Asbestosis
  4. Ischaemia involving right ventricle
  5. Ischaemia involving right ventricle and posterior wall of ventricle
A
  1. Leibmann Sachs endocarditis (SLE) (no heart failure)
How well did you know this?
1
Not at all
2
3
4
5
Perfectly
9
Q
  1. 11.02.32 What is most important factor in myocardial infarction?
  2. Size
  3. Site
  4. Transmural versus subendocardial
  5. Involvement of right ventricle wall
A
  1. Transmural versus subendocardial - higher risk of complications
How well did you know this?
1
Not at all
2
3
4
5
Perfectly
10
Q
  1. 11.03.17 Patient with AMI, delevops chest pain, 5 days later LEAST LIKELY cause is ?
  2. Second AMI
  3. Dresslers syndrome
  4. Bronchopneumonia
  5. PE
  6. Myocardial rupture
A
  1. Dresslers syndrome (onset 2-3 weeks)
  2. 11.03.17 Patient with AMI, develops chest pain, 5 days later LEAST LIKELY cause is? Rob p365
  3. Second AMI
  4. Dresslers syndrome (onset 2-3 weeks)
  5. Bronchopneumonia
  6. PE (thromboembolism occurs with 2 weeks in upto 40%)
  7. Myocardial rupture (occurs anytime in first 2 weeks)

April 2004 – myocardial rupture – peak time of occurrence

How well did you know this?
1
Not at all
2
3
4
5
Perfectly
11
Q
  1. 7.02.20 Man dies from AMI macroscopic pathology shows yellow area in myocardium with granulation tissue at the margin. How old is the infarct?
  2. 3 days
  3. Less than 2 weeks
  4. 2-8 weeks
  5. 8-10 weeks
  6. Months
A
  1. Less than 2 weeks (10days)

7-10 days Maximally yellow-tan and soft, with depressed red-tan margins
Well-developed phagocytosis of dead cells; early formation of fibrovascular granulation tissue at margins

How well did you know this?
1
Not at all
2
3
4
5
Perfectly
12
Q
  1. 11.02.49 Sudden cardiac death is associated with the following except:
  2. Mitral valve prolapse
  3. Mitral stenosis
  4. Aortic stenosis
  5. HOCM
  6. Pulmonary Hypertension
A
  1. Mitral stenosis

SCS: looks like PAH patients get ventricular arrhythmias leading to SCD. Or compresion of LCA by big MPA.

How well did you know this?
1
Not at all
2
3
4
5
Perfectly
13
Q
  1. 11.03.23 In hypertensive heart disease which is LEAST TRUE ?
  2. Circumferential enlargement of left ventricle
  3. Mild ventricular dilatation classically accompanies cardiac enlargement
  4. Mild diastolic filling impairment
  5. Increase in muscle mass more than cardiac size
A
  1. Mild ventricular dilatation classically accompanies cardiac enlargement (concentric hypertrophy of the left ventricle without dilatation and no other possible causitive lesion)
  2. 11.03.23 In hypertensive heart disease which is LEAST TRUE ? Rob p372
  3. Circumfrential enlargement of left ventricle (by definition)
  4. Mild ventricular dilatation classically accompanies cardiac enlargement (concentric hypertrophy of the left ventricle without dilatation and no other possible causitive lesion)
  5. Mild diastolic filling impairment (increased thickness eventually impairs dystolic function)
  6. Increase in muscle mass more than cardiac size (increase in weight disproportionate to the increase in overall size)
How well did you know this?
1
Not at all
2
3
4
5
Perfectly
14
Q
  1. 11.03.06 In rheumatic fever, valve damage is via ?
  2. Cross reaction of antigens to Group B Staph and endocardium
  3. Cross reaction to bacterial glycoproteins in capsule
  4. Entry is by breach in skin and mucous membranes
  5. Initially cysts begin at microscopic level
  6. Direct toxic effect on valve
A
  1. Cross reaction to bacterial glycoproteins in capsule - T - M proteins of certain streptococcal strains induce host antibodies that cross react with tissue glycoproteins in the heart, joints, and other tissues in genetically susceptible individuals (3% of infected patients).
  2. 11.03.06 In rheumatic fever, valve damage is via ? Rob p111, 375-378 (GC)
  3. Cross reaction of antigens to Group B Staph and endocardium - F - group A (beta-haemolytic) strep.
  4. Cross reaction to bacterial glycoproteins in capsule - T - M proteins of certain streptococcal strains induce host antibodies that cross react with tissue glycoproteins in the heart, joints, and other tissues in genetically susceptible individuals (3% of infected patients).
  5. Entry is by breach in skin and mucous membranes - F - acute RF is a hypersensitivity reaction. Onset of symptoms occurs 2-3 wks after original infection, and strep is absent in lesions (the original infection is usually pharyngitis, rarely skin/other infection). Valve (endocardial) involvement in acute RF is the result of fibrinoid necrosis along closure line forming vegetations (verrucae); it is the chronic stage (organisation and fibrosis) that results in valvular damage - leaflet thickening, commisural fusion, ‘fish mouth’ stenoses etc.
  6. Initially cysts begin at microscopic level - F - histologic hallmark is the Aschoff body = central zone of degenerating hypereosinophilic extracellular matrix, with lymphocyte/plasma cell infiltration, plump macrophages (Anitschow cells).
  7. Direct toxic effect on valve - F - see option 3 above.
How well did you know this?
1
Not at all
2
3
4
5
Perfectly
15
Q
  1. 11.02.70 8yr old girl having anaesthetic for MRI to investigate choreiform movements. Murmur heard by anaesthetist most likely due to ?
  2. Previous rheumatic fever
A
  1. Previous rheumatic fever
How well did you know this?
1
Not at all
2
3
4
5
Perfectly
16
Q
  1. 11.03.25 Autopsy on 40 year old male shows abnormal aortic valve with 2 leaflets one of which is large with median raphe, which is TRUE ?
  2. Bicuspid Aortic valve occurs in 0.1% of poplulation
  3. Bicuspid Aortic valve occurs in 1% of poplulation
  4. Patient has rheumatic heart disease
  5. Patient has calcific stenosis
  6. Patient has Marfans
A
  1. Bicuspid Aortic valve occurs in 1% of population - T - 1-2% of the population has bicuspid aortic valve as an isolated abnormality. The two cusps are of unequal size with the larger cusp having a midline raphe. (Robbins)
  2. 11.03.25 Autopsy on 40 year old male shows abnormal aortic valve with 2 leaflets one of which is large with median raphe, which is TRUE ? Rob p379 Big Rob p568 (JS)
  3. Bicuspid Aortic valve occurs in 0.1% of poplulation - F
  4. Bicuspid Aortic valve occurs in 1% of population - T - 1-2% of the population has bicuspid aortic valve as an isolated abnormality. The two cusps are of unequal size with the larger cusp having a midline raphe. (Robbins)
  5. Patient has rheumatic heart disease - F - bicuspid valves are predisposed to infective endocarditis. They are also associated with aortic coarctation, aneurysm and dissection.
  6. Patient has calcific stenosis - F - predisposed to progressive degenerative calcification
  7. Patient has Marfans - F - Marfans is associated with MVP
How well did you know this?
1
Not at all
2
3
4
5
Perfectly
17
Q
  1. 11.03.24 Complications of mitral valve prolapse, which is FALSE ?
  2. Aortic regurgitation secondary to annular dilation
  3. Infective endocarditis
  4. Stroke
  5. Sudden death
  6. Mitral insuffiency
A
  1. Aortic regurgitation secondary to annular dilation - F - this is see in Marfans with loss of medial support results in progressive dilatation of the aortic ring and root of the aorta, giving rise to severe aortic incompetance
  2. 11.03.24 Complications of mitral valve prolapse, which is FALSE ? Rob p379 (JS)
  3. Aortic regurgitation secondary to annular dilation - F - this is see in Marfans with loss of medial support results in progressive dilatation of the aortic ring and root of the aorta, giving rise to severe aortic incompetance
  4. Infective endocarditis - T - manyfold more frequent than in general population. Overall only 3% of patients with MVP develop complications.
  5. Stroke - T - emboli of leaflet or atrial wall thrombi can cause strokes or systemic infarcts.
  6. Sudden death - T - uncommon but due to ventricular or atrial arrhythmias
  7. Mitral insuffiency - T - due to leaflet deformity, dilation of the annulus or cordal lengthening or sudden onset due to cordal rupture.
How well did you know this?
1
Not at all
2
3
4
5
Perfectly
18
Q
  1. 11.03.19 Patient with large exophytic mass on cardiac valve referred for CT:
  2. If has splenic lesions is most likely Candida
  3. If has non dilated cardiomyopathy is most likely carcinoid
  4. If has pericardial effusion is likely Staph with abscess
  5. If has thromboemboli is unlikely to be marantic endocarditis
  6. If has pulmonary infiltrates patient may have Wegners
A
  1. If has pericardial effusion is likely Staph with abscess - T
    * LW: probably most correct: Valvular vegetations can erode into underlying myocardium to produce ring abscess. Can be associated with pericardial effusion.

[Pretty limited info to go on in question stem, likely incomplete recall]

  1. 11.03.19 Patient with large exophytic mass on cardiac valve referred for CT: Rob p381 (–)
  2. If has splenic lesions is most likely Candida (Streptococci and staphylococci were the causative organisms in 85%):
    * LW: could be true, as large valvular lesions are usually fungal.
  3. If has non dilated cardiomyopathy is most likely carcinoid: *LW False, right sided valvular disease.
  4. If has pericardial effusion is likely Staph with abscess - T -
  5. If has thromboemboli is unlikely to be marantic endocarditis: *LW - False: Marantic endocarditis commonly associated with thromboemboli. (frequently occurs concomitantly with venous thromboses or pulmonary embolism, suggesting a common origin in a hypercoagulable state with systemic activation of blood coagulation such as disseminated intravascular coagulation)
  6. If has pulmonary infiltrates patient may have Wegners: False - (most likely IV drug user - S. aureus, Candida parapsilosis and C. albicans)
How well did you know this?
1
Not at all
2
3
4
5
Perfectly
19
Q
  1. 11.03.22 Patient with restrictive cardiomyopathy, LEAST LIKELY finding on Chest CT is ?
  2. Basal bullous change
  3. Radiotherapy changes
  4. Bilateral hilar lymphadenopathy
  5. Multiple pulmonary nodules
  6. Bilateral bronchiectasis
A
  1. Basal bullous change - F - This is not typical of any of the causes of restrictive cardiomyopathy. Restrictive cardiomyopathy is characterised by decreased ventricular compliance, resulting in impaired filling during diastole. It can be idiopathic or associated with distinct diseases that affect the myocardium, principally radiation fibrosis, amyloidosis, sarcoidosis, metastatic tumor, or products of inborn errors of metabolism.
  2. 11.03.22 Patient with restrictive cardiomyopathy, LEAST LIKELY finding on Chest CT is ? Rob p387 (JS)
  3. Basal bullous change - F - This is not typical of any of the causes of restrictive cardiomyopathy. Restrictive cardiomyopathy is characterised by decreased ventricular compliance, resulting in impaired filling during diastole. It can be idiopathic or associated with distinct diseases that affect the myocardium, principally radiation fibrosis, amyloidosis, sarcoidosis, metastatic tumor, or products of inborn errors of metabolism.
  4. Radiotherapy changes - T - radiation fibrosis
  5. Bilateral hilar lymphadenopathy - T- sarcoidosis
  6. Multiple pulmonary nodules - T - metastatic tumour/sarcoid.
  7. Bilateral bronchiectasis - T - amyloidosis secondary to chronic lung disease/ sarcoid- traction bronchiectasis from fibrosis.
How well did you know this?
1
Not at all
2
3
4
5
Perfectly
20
Q
20.	Sep03.22 4 y.o. in ICU with cardiomegaly. Discharged, then one month later – left atrial enlargement. Cause?
Kawasaki
Takayasu
Rheumatic fever
Subacute bacterial Endocarditis
A
  1. Sep03.22 4 y.o. in ICU with cardiomegaly. Discharged, then one month later – left atrial enlargement. Cause?
    Rheumatic fever
  • mitral valve involved by itself 70% of the time, with mitral and aortic valves involved 25% of the time
  • chronic mitral stenosis leads to left atrial hypertrophy

SCS Added options.

How well did you know this?
1
Not at all
2
3
4
5
Perfectly
21
Q
  1. Sep03.24 Atrial myxoma – atypical feature.
  2. 3cm papillary soft tissue mass sessile
  3. 8 cm globular mass pedunculated
  4. atrial septum
  5. can prolapse into ventricle
  6. more common in the right atrium
A
  1. more common in the right atrium - F - left to right ratio is 4:1 with 90% located in the atria
  2. Sep03.24 Atrial myxoma – atypical feature. (JS)
  3. 3cm papillary soft tissue mass sessile - T - sessile or pedunculated masses that vary from hard globular masses mottled with haemorrhage to soft, translucent, papillary or villous lesions having a gelatinous appearance
  4. 8 cm globular mass pedunculated - T - size range is 1 to 10cm and they are usually solitary
  5. atrial septum - T - fossa ovalis in the atrial septum is a common site
  6. can prolapse into ventricle - T - pedunculated form mobile enough to move into or through the AV valves during systole causing intermittent and often position dependent obstruction or “wrecking ball” effect
  7. more common in the right atrium - F - left to right ratio is 4:1 with 90% located in the atria
How well did you know this?
1
Not at all
2
3
4
5
Perfectly
22
Q
  1. Sep03.35 Mitral ring calcification
  2. Soft
  3. hard, MVR & conduction defects
  4. normal variant
A
  1. hard, MVR & conduction defects
  • USUALLY ASYMPTOMATIC
  • THIS COULD BE BAD RECALL, CAREFUL IF SEE IT IN EXAM
  • Degenerative calcific deposits can develop in the ring ( annulus) of the mitral valve, visualized on gross inspection as irregular, stony hard, and occasionally ulcerated nodules (2 to 5 mm in thickness) that lie behind the leaflets
  • The process generally does not affect valvular function.
  • In unusual cases, however, it may lead to regurgitation by interfering with systolic contraction of the mitral valve ring, to stenosis by impairing opening of the mitral leaflets, or to arrhythmias and occasionally sudden death by the calcium deposits penetrating sufficiently deeply to impinge on the AV conduction system.
  • Because ulcerated calcific nodules may provide a site for thrombi that can embolize, some patients with mitral annular calcification have an increased risk of stroke.
  • The calcific nodules can also be the nidus for infective endocarditis.
  • Heavy calcific deposits are sometimes visualized on echocardiography or seen as a distinctive, ringlike opacity on chest radiographs.
  • Mitral annular calcification is most common in women over 60 years of age and individuals with myxomatous mitral valve (see later) or elevated left ventricular pressure (as in systemic hypertension, aortic stenosis, or hypertrophic cardiomyopathy).
How well did you know this?
1
Not at all
2
3
4
5
Perfectly
23
Q
  1. Sep03.68 Pericardial effusion on ultrasound. Which is the least likely cause?
  2. Cocksackie virus
  3. Sarcoid
  4. Uraemia
  5. Rheumatoid arthritis
A
  1. Sarcoid
How well did you know this?
1
Not at all
2
3
4
5
Perfectly
24
Q
  1. 11.03.26 In secundum ASD which is MOST TRUE ?
  2. Accounts for 60-70% of ASD’S
  3. Occurs in artioventricular valve
  4. Usually isolated anomaly
  5. Associated with left ventricular enlargement
  6. Present in neonatal period
A
  1. Usually isolated anomaly (Most are isolated (not associated with other anomalies))
  2. 11.03.26 In secundum ASD which is MOST TRUE ? Rob p388
  3. Accounts for 60-70% of ASD’S (90% of all ASDs)
  4. Occurs in artioventricular valve (fenestrated oval fossa near the mid-septum - valve or limbus of the fossa ovalis)
  5. Usually isolated anomaly (Most are isolated (not associated with other anomalies))
  6. Associated with left ventricular enlargement (When associated with another defect, such as tetralogy of Fallot, the other defect is usually hemodynamically dominant.)
  7. Present in neonatal period (most do not become evident till age 30)
  • The secundum ASD, accounting for approximately 90% of all ASDs, comprises a defect located at and results from a deficient or fenestrated oval fossa near the mid-septum.
  • Most are isolated (not associated with other anomalies).
  • When associated with another defect, such as tetralogy of Fallot, the other defect is usually hemodynamically dominant.
  • The atrial aperture may be of any size and may be single, multiple, or fenestrated
How well did you know this?
1
Not at all
2
3
4
5
Perfectly
25
Q
  1. 7.02.30 Carcinoid of the heart ?
A
  1. 7.02.30 Carcinoid of the heart ? Big Rob p577
    • Cardiac involvement
    o Pulmonic and tricuspid valve thickening and stenosis
    o Endocardial fibrosis, principally in the right ventricle
    o (Bronchial carcinoids affect the left side)
  • Carcinoid tumors of the ileum are the most likely to metastasize, with involvement of the regional lymph nodes and liver.
  • Usually only carcinoid tumors that invade the liver result in carcinoid heart disease.
  • The cardiac lesions may be related to large circulating quantities of serotonin, bradykinin, or other substances secreted by the tumor, which usually are inactivated by the liver, lungs, and brain.
  • Hepatic metastases apparently allow large quantities of tumor products to reach the heart.
  • The preferential right-sided involvement presumably is related to inactivation of the offending humoral substance(s) by the lungs.
  • In 5 to 10 percent of cases, significant left-sided valvular disease develops, related in most to passage of blood directly from the right to the left side of the heart through a patent foramen ovale, or less commonly by tumor involvement of the lungs
  • Physical examination usually reveals a systolic murmur along the left sternal border, produced by tricuspid regurgitation; in some cases, there may be a concomitant murmur of pulmonic stenosis and/or regurgitation
  • The chest roentgenogram is normal in half of the patients, but it may reveal enlargement of the heart and pleural effusions or nodules; the pulmonary artery trunk is typically of normal size, without evidence of poststenotic dilatation as occurs in congenital pulmonic stenosis
  • The hemodynamic findings most commonly encountered are those of tricuspid regurgitation and occasionally pulmonic stenosis.
How well did you know this?
1
Not at all
2
3
4
5
Perfectly
26
Q
  1. Sep03.80 Patient suffering from transient myocardial depression after AMI for one week:
  2. stunned myocardium
  3. cardiogenic shock
A
  1. stunned myocardium - T - abnormalities in cellular biochemistry may persist for several days after ischaemia and lead to a noncontractile state. Such stunning can produce a state of transient reversible cardiac failure that may require pump assistance to support the pt until cardiac fxn returns.
  2. Sep03.80 Patient suffering from transient myocardial depression after AMI for one week: (GC)
  3. stunned myocardium - T - abnormalities in cellular biochemistry may persist for several days after ischaemia and lead to a noncontractile state. Such stunning can produce a state of transient reversible cardiac failure that may require pump assistance to support the pt until cardiac fxn returns.
  4. cardiogenic shock - F - occurs in 10-15% of patients after an AMI, generally with a large infarct (>40% of LV); 70% mortality rate.
How well did you know this?
1
Not at all
2
3
4
5
Perfectly
27
Q
  1. PATH2004 Which of the following would be an UNEXPECTED finding following coronary angioplasty?
  2. Luminal expansion
  3. Plaque rupture
  4. Thickened intact intima
  5. Medial dissection
  6. Proliferative restenosis in 30-50% of patients at 6 months
A
  1. Thickened intact intima - F - The split encompasses the intima and media
  2. PATH2004 Which of the following would be an UNEXPECTED finding following coronary angioplasty? (JS)
  3. Luminal expansion - T - The key elements of luminal expansion in angioplasty are plaque rupture, medial dissection and stretching of the media of the dissected segment.
  4. Plaque rupture - T
  5. Thickened intact intima - F - The split encompasses the intima and media
  6. Medial dissection - T
  7. Proliferative restenosis in 30-50% of patients at 6 months - T - The long-term success of angioplasty is limited by the development of proliferative restenosis that occurs in approximately 30 to 50% of patients within the first 4 to 6 months after angioplasty
How well did you know this?
1
Not at all
2
3
4
5
Perfectly
28
Q
  1. PATH2004 AMI, develops chest pain, 5 days later LEAST LIKELY cause is ?
A
  1. PATH2004 AMI, develops chest pain, 5 days later LEAST LIKELY cause is ?
•	myocardial rupture (2.5%) 
o	occurs in 10% of patients dying in hospital from acute infarct 
o	occurs any time first 2 weeks 
o	most common between 4 and 7 days 
o	develops over several days 

• septal rupture

• acute pericarditis at 2 to 3 days (nearly 100% of transmural infarcts)
o acute pericarditis in 15% within 2 to 4 days

• papillary muscle rupture (1%)
o day 3 -> acute left ventricular failure -> high mortality rate

• Not Ventricular aneurysm
o This is a late complication that most commonly results from a large transmural anteroseptal infarct

• Not Dressler syndrome
o Onset 2-3 weeks

How well did you know this?
1
Not at all
2
3
4
5
Perfectly
29
Q
  1. PATH2004 Regarding acute rheumatic fever, which of the following is the LEAST CORRECT:
  2. It follows pharyngeal infection with group A streptococcus
  3. Serofibrinous pericarditis is common
  4. Acute arthritis typically affects large joints like the knees
  5. Aschoff bodies are an uncommon histological feature of subcutaneous nodules
  6. Rheumatic pneumonitis is a recognised rare complication
A
  1. Aschoff bodies are an uncommon histological feature of subcutaneous nodules - F - Aschoff bodies are foci of fibrinoid degeneration surrounded by lymphocytes, plasma cells and macrophages and can be found in any of the three layers of the heart
  2. PATH2004 Regarding acute rheumatic fever, which of the following is the LEAST CORRECT: (JS)
  3. It follows pharyngeal infection with group A streptococcus - T - acute immunologically mediated multisystem inflammatory disease that occurs a few weeks after an episode of group A streptococcal pharyngitis. Major manifestations are migratory polyarthritis, carditis, subcutaneous nodules, erythema marginatum, Sydenham chorea.
  4. Serofibrinous pericarditis is common - T - causes a pancarditis with a serofibrinous pericardial exudate (bread and butter pericarditis), a myocarditis and endocardial involvement
  5. Acute arthritis typically affects large joints like the knees - T - migratory polyarthritis involves the large joints
  6. Aschoff bodies are an uncommon histological feature of subcutaneous nodules - F - Aschoff bodies are foci of fibrinoid degeneration surrounded by lymphocytes, plasma cells and macrophages and can be found in any of the three layers of the heart
  7. Rheumatic pneumonitis is a recognised rare complication T - diffuse pulmonary consolidation and tachypnoea, unresponsive to antibiotic or steriod therapy in a patient with active Rheumatic fever
  • rheumatic fever occurs in 3% of patients after infection with group A streptococcus
  • pericardium - fibrinous or serofibrinous exudate (bread and butter pericarditis) which generally resolves without sequelae
  • migratory large joint arthritis
  • Aschoff bodies. They constitute foci of fibrinoid degeneration surrounded by lymphocytes (primarily T cells), occasional plasma cells, and plump macrophages called Anitschkow cells (pathognomonic for rheumatic fever). These distinctive cells have abundant amphophilic cytoplasm and central round-to-ovoid nuclei in which the chromatin is disposed in a central, slender, wavy ribbon (hence the designation caterpillar cells).
  • In cases of acute rheumatic fever with severe carditis, areas of patchy pneumonitis are sometimes seen. Many observers believe that these pulmonary infiltrates represent a specific rheumatic pneumonia
How well did you know this?
1
Not at all
2
3
4
5
Perfectly
30
Q
  1. PATH2004 Regarding the macroscopic appearance of acute MI, which of the following is the most correct:
  2. The left circumflex coronary shows severe stenosing atherosclerosis in 40-50% of cases
  3. Isolated infarct of the right ventricle occurs in 1-3% of cases
  4. The median time to rupture is 2-3 days
  5. Pericarditis usually develops at 24hrs post event
  6. Infarcts <48hrs old are usually inapparent on gross examination
A
  1. Isolated infarct of the right ventricle occurs in 1-3% of cases
  2. PATH2004 Regarding the macroscopic appearance of acute MI, which of the following is the most correct:
  3. The left circumflex coronary shows severe stenosing atherosclerosis in 40-50% of cases
  4. Isolated infarct of the right ventricle occurs in 1-3% of cases - True (word for word from Robbins>
  5. The median time to rupture is 2-3 days
  6. Pericarditis usually develops at 24hrs post event
  7. Infarcts <48hrs old are usually inapparent on gross examination

Location
• 95% involve a portion of the left ventricle
• LAD (45%)
o anterior wall of left ventricle near apex, anterior 2/3’s of interventricular septum

• RCA (35%)
o inferior/posterior wall of left ventricle, posterior 1/3 of IV septum, posterior right ventricular free wall in some cases

• Left circumflex (15%)
o lateral wall of the left ventricle
• Isolated infarction of the right ventricle, however, occurs in only 1 to 3%

Myocardial Rupture (2.5%)
• occurs in 10% of patients dying in hospital from acute infarct
• occurs any time first 2 weeks, most common between 4 and 7 days

Pericarditis
• 2 to 3 days (nearly 100% of transmural infarcts) acute pericarditis in 15% within 2 to 4 days

Dresslers syndrome (autoimmune pericarditis) (2%) onset 2-3 weeks 
Gross morphology 
•	Dark mottling may present by 12 hrs, definitely by 24hrs
How well did you know this?
1
Not at all
2
3
4
5
Perfectly
31
Q
  1. PATH2004 Regarding infective endocarditis, which of the following is the LEAST CORRECT:
  2. Right sided heart valves in 50% of narcotic related cases
  3. Diffuse glomerulonephritis seen in 50% of untreated cases
  4. Positive blood cultures are seen in 85-90% of cases
  5. Strep pneumonia is the leading cause of acute endocarditis
  6. With mechanical prostheses infections are usually located on the margin of the sewing ring
A
  1. Strep pneumonia is the leading cause of acute endocarditis – F – (note that path notes say more than ½ cases due to various streptococci – however most commonly gingival disease) – since 1980s IE due to S aureus (primary pathogen) primary risk factor is intravascular devices (eMed).
  2. PATH2004 Regarding infective endocarditis, which of the following is the LEAST CORRECT: (TW)
  3. Right sided heart valves in 50% of narcotic related cases – T - in 75% of IVDA IE, no underlying valvular abnormalities are noted, and 50% of these infections involve the tricuspid valve (eMed).
  4. Diffuse glomerulonephritis in 50% of untreated cases – T ~30% UpToDate
  5. Positive blood cultures in 85-90% of cases – T – path notes / UpToDate (but varies depending on series and geography)/ eMed
  6. Strep pneumonia is the leading cause of acute endocarditis – F – (note that path notes say more than ½ cases due to various streptococci – however most commonly gingival disease) – since 1980s IE due to S aureus (primary pathogen) primary risk factor is intravascular devices (eMed).
  7. With mechanical prostheses infections are usually located on the margin of the sewing ring – T – endocarditis is located at the prosthesis-tissue intervace, causing a ring intervace (Robbins).
  • Endocarditis of native but previously damaged or otherwise abnormal valves is caused most commonly (50 to 60% of cases) by alpha-hemolytic (viridans) streptococci
  • Staphylococcus aureus organisms commonly found on the skin can attack either healthy or deformed valves; they are responsible for 10 to 20% of cases overall and are the major offender in intravenous drug abusers.
  • The roster of the remaining bacteria includes enterococci and the so-called HACEK group ( Haemophilus, Actinobacillus, Cardiobacterium, Eikenella, and Kingella), all commensals in the oral cavity
  • Prosthetic valve endocarditis is caused most commonly by coagulase-negative staphylococci (e.g., Staphylococcus epidermidis).
  • Other agents causing endocarditis include gram-negative bacilli and fungi. In about 10% of all cases of endocarditis, no organism can be isolated from the blood ( culture-negative endocarditis) because of prior antibiotic therapy, other difficulties in isolation of the offending agent, or deeply embedded organisms within the enlarging vegetation are not released into the blood.

How well did you know this?
1
Not at all
2
3
4
5
Perfectly
32
Q
  1. Bowel Ischaemia:
    a. Can be related to superior mesenteric vein thrombosis
    b. Is associated with paroxysmal nocturnal haemoglobinuria
    c. Vasospasm indicates underlying atherosclerosis and stenosis
    d. Occlusion is typically 10cm distal to the SMA origin
    e. Plain film changes are only seen after bowel infarction
A

a. Can be related to superior mesenteric vein thrombosis - T - venous occlusion <10%. SMV > IMV > PV.
b. Is associated with paroxysmal nocturnal haemoglobinuria - T - PNH is a disorder characterised by a defect in the GPI anchor due to an abnormality in th PIG-A gene. Result in abnormalities in hemopoietic function, including hemolytic anemia, a hypercoagulabel state, and diminished hematopoiesis.

  1. Bowel Ischaemia: (TW)
    a. Can be related to superior mesenteric vein thrombosis - T - venous occlusion <10%. SMV > IMV > PV.
    b. Is associated with paroxysmal nocturnal haemoglobinuria - T - PNH is a disorder characterised by a defect in the GPI anchor due to an abnormality in th PIG-A gene. Result in abnormalities in hemopoietic function, including hemolytic anemia, a hypercoagulabel state, and diminished hematopoiesis.
    c. Vasospasm indicates underlying atherosclerosis and stenosis - F
    d. Occlusion is typically 10cm distal to the SMA origin - F - SMA thrombosis (20-40%) at origin + site of atherosclerotic narrowing. Embolus (40-50%) just distal to middle colic artery.
    e. Plain film changes are only seen after bowel infarction - F - ischaemia, oedematous bowel, thumb printing. Could also have bowel obstruction as cause of ischaemia (pre-infarction).
How well did you know this?
1
Not at all
2
3
4
5
Perfectly
33
Q
  1. Aortic Stent Grafts:
    a. Sac size greater than 8 cm is a contra indication
    b. The neck of the aneurysm is defined as the distance from the inferior most renal artery to the start of the aneurysm
    c. Immediate endoleak always needs further intervention
    d. Retro leak from lumbar arteries needs further intervention
    e. Luminal diameter is best measured via angiography with a graded catheter
A

b. The neck of the aneurysm is defined as the distance from the inferior most renal artery to the start of the aneurysm - T - the proximal neck is defined as the segment of aorta between the origin of the lowest renal artery and superior aspect of the aneurysm. Most endographs require a length of greater than 15mm to ensure adequate proximal seal.

  1. Aortic Stent Grafts: (TW)
    a. Sac size greater than 8cm is a contraindication - F - for example the Zenith AAA grafts range from 82mm to 149mm in length.
    b. The neck of the aneurysm is defined as the distance from the inferior most renal artery to the start of the aneurysm - T - the proximal neck is defined as the segment of aorta between the origin of the lowest renal artery and superior aspect of the aneurysm. Most endographs require a length of greater than 15mm to ensure adequate proximal seal.
    c. Immediate endoleak always needs further intervention - F - depends on type. type I endoleak (ie incompetent seal at either proximal or distal attachment site) must be repaired as soon as possible. Type III endoleaks represtn flow into sac from separation between components of a modular system, or tears in the endograft - these are repaired with an additional endograft. Type IV leaks are due to egress of blood through the pores of the fabric - these heal spontaneously. Type II enodleak (most common) - management is controversial. Some argue that as spontaneous resolution occurs 30-100% of cases, a “wait and see” approach is preferable. UTD recommendations are occluding type II endoleaks that have not spontaneously thrombosed in one month (as systemic pressures have been noted in aneuyrsm sac in the presence of type II endoleaks
    d. Retro leak from lumbar arteries needs further intervention - ?T/F - controversial, see ans c.
    e. Luminal diameter is best measured via angiography with a graded catheter - ?F
How well did you know this?
1
Not at all
2
3
4
5
Perfectly
34
Q
  1. Regarding Cervico Cephalic Vessel Dissection: which is false?

a. Causes 5% to 20% of CVAs in young to middle aged patients
b. Has a rim of T1 hyper intensity surrounding a void in the vessels
c. Imaging identifies vasculopathy in the majority
d. The location of vertebral artery dissections is most often proximal (between C2 and C6)
e. The location of an ICA dissection is most often just immediately distal to the bulb

A

d. The location of vertebral artery dissections is most often proximal (between C2 and C6) - F - at the level of C1/2 in 65% [Dahenrt]. ie. predominantly located in the pars transversaria (V2, 35%) or the atlas loop (V3, 34%) [RG]. Bilateral vertebal artery dissections in 5%.

c. Imaging identifies vasculopathy in the majority: *LW: vasculopathy in terms of atherosclerosis is not a risk factor for dissection, so hence not commonly found, would favour this to also be false.
* *LJS - agree

  1. Regarding Cervico Cephalic Vessel Dissection: (GC)
    a. Causes 5% to 20% of CVAs in young to middle aged patients - T - 5-20% in those aged 40-60yrs [Dahnert]; up to 1/4 of strokes,with a peak prevalence in the 5th decade [RG].

b. Has a rim of T1 hyperintensity surrounding a void in the vessels - T - subacute hematoma (7days-2mths) on T1 + fat sat appears as a bright crescent-shaped area around an eccentric flow void. Note that in the early and chronic stage, the haematoma is usually isointense to surrounding structures on T1.
d. The location of vertebral artery dissections is most often proximal (between C2 and C6) - F - at the level of C1/2 in 65% [Dahenrt]. ie. predominantly located in the pars transversaria (V2, 35%) or the atlas loop (V3, 34%) [RG]. Bilateral vertebal artery dissections in 5%.
e. The location of an ICA dissection is most often just immediately distal to the bulb - T - extracranial ICA dissection affects the cervical part of the artery distal to the carotid bulb and tends not to extend beyond its entry into the petrous portion. [Craniocervical a. dissection, RG 2008]

How well did you know this?
1
Not at all
2
3
4
5
Perfectly
35
Q
  1. Fixed inferior defects in a thallium scan are caused by: which one is false
    a. Diaphragmatic attenuation b. Mastectomy c. Right coronary artery infarct d. Pacemaker leads e. Left bundle branch block
A

b. Mastectomy - F - implants could cause attentuation artifact.

  1. Fixed inferior defects in a thallium scan are caused by: (GC)
    a. Diaphragmatic attenuation - T - variation in tracer intensity by 15-20% btn regions on planar images may be normal (due to soft tissue attentuation artifacts from subdiaphragmatic abdominal contents or breast tissue).
    b. Mastectomy - F - implants could cause attentuation artifact.
    c. Right coronary artery infarct - T - RCA territory is 5-8 o’clock on the short axis (oblique coronal).
    d. Pacemaker leads - T - attentuation artifact.
    e. Left bundle branch block - ?T
How well did you know this?
1
Not at all
2
3
4
5
Perfectly
36
Q
  1. With regard to cerebral aneurysms: (multiple true)
    a. In children are associated with trauma and infection
    b. Multiple aneurysms are strongly associated with male gender
    c. A truly azygos anterior cerebral artery is associated with lobar holoprosencephaly and saccular aneurysms
    d. Are multiple in 15% to 20%
    e. Occur in the posterior circulation in 40%
A

there are 3 trues : a, c, d

  1. With regard to cerebral aneurysms: (TW)
    a. In children are associated with trauma and infection - T - Aneuyrsms in children may be congenital or idiopathic (77%), inflammatory or mycotic (11%), or traumatic (11%).

b. Multiple aneurysms are strongly associated with male gender - F - Female > Male (Neuroradiology 2004). Female preponderance has been reported in up to 70% of multiple aneurysms and a 2.4:1 ratio of females > males was reported in the Cooperative Study of Intracranial Aneurysms and SAH.
c. A truly azygos anterior cerebral artery is associated with lobar holoprosencephaly and saccular aneurysms - T - azygos ACA is a rare variation of the COW in which the A2 segments of both ACAs are represented by a singl common vessel from which arise all the major vessels supplying most of both anterior cerebral hemispheres, as well as the corpus callosum (ie a true azygos). Associated with holoprosencephaly. High indicence of berry aneurysms. Abnormal medial and intimal elements in the vessel wall of this developmental variant may account for the increased prevalence of associated saccular aneurysms.
d. Are multiple in 15% to 20% - T

e. Occur in the posterior circulation in 40% - F - 90-95% of aneurysms arise from COW
- 90% from anterior circulation, 1
0% from posterior circulation.
ACom 30-35%,
PCom 30-35%,
MCA 20-30%, B
asilar apex 5-10%, other posterior 1-3%.

Variations of the distal ACA: trus azygos artery, from which all major branches are given off to both cerebral hemispheres; a bihemispheric ACA, where both right and left ACAs are present, but one is rudimentary and most of the major branches to both hemispheres arise from the other ACA; a triple ACA, with the accessory ACA arising from the anterior communcating artery.

How well did you know this?
1
Not at all
2
3
4
5
Perfectly
37
Q
  1. Cerebral angiography: which is false?
    a. The supraclinoid internal carotid artery is best seen in a caudal (Waters) projection
    b. The origin of the opthalmic artery is intradural in the majority of people
    c. Internal carotid injections are of 8mls at 2mls per second
    d. Cavernous haemangiomata are angiographically occult
    e. The enhancement of meningiomas persist late into the venous phase
A

a. The supraclinoid internal carotid artery is best seen in a caudal (Waters) projection - ?F -

Cerebral angiography: (TW)
a. The supraclinoid internal carotid artery is best seen in a caudal (Waters) projection - ?F - ICA/ophthalmic: transorbital oblique (15-30 degree rotation, and 7 degrees cephalad); Caldwell’s oblique (15-30 degrees rotation, and 7 degrees caudad). Waters is ~30 degrees caudad?

b. The origin of the opthalmic artery is intradural in the majority of people - T - The origin of the ophthalmic artery is intradural in approx 90% of anatomic dissections (Diagnostic cerebral angiography, AG Osborn).
c. Internal carotid injections are of 8mls at 2mls per second - T - CCA 4-5ml/s for 2s (ie total 8-10mls)
d. Cavernous haemangiomata are angiographically occult - T - cavernous malformation: DSA is usually normal “angiographically occult vascular malformation”

e. The enhancement of meningiomas persist late into the venous phase - T - mother-in-law - “comes early, stay late”
- alternate memory aid to remember big nando: the pre-mature ejaculator who wants to cuddle afterwards.

How well did you know this?
1
Not at all
2
3
4
5
Perfectly
38
Q
  1. Which of the following statements with regards to fatty plaques?(streaks?) is LEAST CORRECT?
  2. Moreover, they frequently affect individuals in geographic locales and populations in which atherosclerotic plaque is uncommon.
  3. ?% of kids that have them
  4. ?site
  5. bld vess and heart pg 516 -517 robbins.
A

A is right if it is streak

can occur in all children even poor skinny ones
involve 4-5% of aorta by 10yo

How well did you know this?
1
Not at all
2
3
4
5
Perfectly
39
Q

Regarding polyarteritis nodosa. Which of the following is/are LEAST CORRECT:

a. The kidneys are most commonly involved in autopsy
a. Aneurysms are seen at angiography in 50%
b. 70% are seropositive for Hepatitis B surface antigen
c. Churg Strauss syndrome frequently involves pulmonary and splenic vessels
d. Serum anti-neutrophil antibody titres correlate with disease activity

A

2 and 4 false

  1. Regarding polyarteritis nodosa. Which of the following is/are false: (GC)
  2. The kidneys are most commonly involved in autopsy - T - vessels of the kidneys, heart, liver, and GIT are involved in descending order.
  3. Aneurysms are seen at angiography in 50% - T - multiple small intrarenal microaneurysms typically at branchpoints in 50-60% - hallmark feature. Lesions usually involve only part of the vessel circumference; the inflammatory process weakens the arterial wall and can lead to aneurysms or even rupture.
  4. 70% are seropositive for Hepatitis B surface antigen - F - 30% have hepatitis B antigenaemia, and HBsAg-HBsAb immune complexes can be demonstrated in the lesions.
  5. Churg Strauss syndrome frequently involves pulmonary and splenic vessels - T - aka allergic granulomatosis and angiitis; vascular lesions can resemble PAN and microscopic polyangiitis, but in the lung, heart, spleen, peripheral nerves and skin there are also intra- and extravascular granulomas. PAN spares the pulmonary circulation.
  6. Serum anti-neutrophil antibody titres correlate with disease activity - F - Adelaide path notes: p-ANCA is often present in serum and correlates with disease activity. *LW: I favour false per Robbins.
    * *WJI disagree with rationale. PAN is ANCA negative. So still false.
How well did you know this?
1
Not at all
2
3
4
5
Perfectly
40
Q
  1. Which of the following would be an UNEXPECTED finding following coronary angioplasty?
    a. Luminal expansion
    b. Plaque rupture
    c. Thickened intact intima
    d. Medial dissection
    e. Proliferative restenosis in 30-50% of patients at 6 months
A
  1. Thickened intact intima - F - The split encompasses the intima and media
  2. Which of the following would be an UNEXPECTED finding following coronary angioplasty? (JS)
  3. Luminal expansion - T - The key elements of luminal expansion in angioplasty are plaque rupture, medial dissection and stretching of the media of the dissected segment.
  4. Plaque rupture - T
  5. Thickened intact intima - F - The split encompasses the intima and media
  6. Medial dissection - T
  7. Proliferative restenosis in 30-50% of patients at 6 months - T - The long-term success of angioplasty is limited by the development of proliferative restenosis that occurs in approximately 30 to 50% of patients within the first 4 to 6 months after angioplasty
How well did you know this?
1
Not at all
2
3
4
5
Perfectly
41
Q
  1. Which of the following conditions IS TRUE with regards to hypertension?
  2. 5-10% of dissections have no obvious intimal tear - true
A
  1. Which of the following conditions IS TRUE with regards to hypertension?
  2. 5-10% of dissections have no obvious intimal tear - true
How well did you know this?
1
Not at all
2
3
4
5
Perfectly
42
Q
  1. Focal renal infarct, least likely:
    a. Aortic atherosclerosis
    b. Mitral valve regurgitation
    c. Post AMI
    d. PAN
    e. Adult Haemolytic Uremic Syndrome
    f. Candida endocarditis
A
  1. Mitral valve regurgitation - F - MR is not a cause of embolism in itself, but could cause embolic renal infarction if the regurg is due to infective endocarditis.
    * *LW: Mitral valve regurgitation by itself would not cause renal infarction, while HUS will cause microthrombi within renal vasculature, thus is more likely than isolated MR. Hence preferred answer is MR
  2. Focal renal infarct, least likely: (GC)
  3. Aortic atherosclerosis - T - thromboembolism
  4. Mitral valve regurgitation - F - MR is not a cause of embolism in itself, but could cause embolic renal infarction if the regurg is due to infective endocarditis.
  5. Post AMI - T - LV mural thrombus
  6. PAN - T - kidney involved in 70-90%. Focal panmural necrotising vasculitis – mucoid degeneration & fibrinoid necrosis begins in media – intimal proliferation & thrombosis.
  7. Adult Haemolytic Uremic Syndrome - ? T/F - similar to childhood (classic) HUS but seen in association with:
    • Infection – verocytotoxic E.coli (O157:H7), shigella, viral
    • Postpartum
    • 2° HUS – scleroderma, SLE, malignant HTN, drugs etc.

Classic triad:
4. Thrombocytopaenia
5. Microangiopathic haemolytic anaemia
6. Acute renal failure
Path: capillary & endothelial injury – mechanical damage to RBCs – formation of hyaline thrombi (predominantly glomeruli, but may extend into arterioles & larger arteries if severe) – cortical necrosis (fleabitten).
Several MCQ teams have answered that HUS is False as it causes cortical necrosis and not a focal renal infarct. Dahnert says focal infarction DOES occur.
6. Candida endocarditis - T - embolism

Causes of renal infarction:
Most due to embolism
Major source of emboli = mural thrombi from LA & LV from AMI
Less common sources
• vegetative endocarditis
• thrombosis in aortic aneurysms and aortic atherosclerosis
Rare causes
• thrombosis in advanced atherosclerosis
• acute vasculitis of polyarteritis nodosa
[Robbins, Adelaide path notes, Dahnert pg 929]

How well did you know this?
1
Not at all
2
3
4
5
Perfectly
43
Q

. Least likely site for hypertensive bleed in the brain is:

a. hippocampus
b. cerebellum
c. basal ganglia
d. thalamus
e. putamen
f. brainstem

A
  1. hippocampus
How well did you know this?
1
Not at all
2
3
4
5
Perfectly
44
Q

. Cavernous Angioma – What is not typical

a. detectable at angiography
b. Bleeding tendency
c. No intervening brain
d. Pseudo capsule + surrounding hemosiderin laden macrophages
e. Associated venous angioma

A
  1. detectable at angiography F - angiographically occult lesions
  2. Cavernous Angioma – What is not typical (JS)
  3. detectable at angiography F - angiographically occult lesions
  4. Bleeding tendency - T - contain blood degredation products of varying ages, haemorrhage risk is 0.5-1% per year but occult bleeds are more common (Osborne)
  5. No intervening brain - T - dilated endothelial cell-lined spaces with no normal brain within the lesion
  6. Pseudo capsule + surrounding hemosiderin laden macrophages - T - A low signal haemosiderin ring is typical on MR
  7. Associated venous angioma - T - up to 1 third of DVAs are associated with cavernous angiomas (Osborne)
How well did you know this?
1
Not at all
2
3
4
5
Perfectly
45
Q
  1. Wegener’s granulomatosis is characterised by all of the following except
  2. Generalised vasculitis:
  3. GN
  4. Granulomatosis of the upper and lower respiratory tract
  5. ANA +ve
  6. ANCA +ve
A
  1. ANA +ve - F 18.
  2. Wegener’s granulomatosis is characterised by all of the following except (JS)
  3. generalised vasculitis - T - A necrotising vasculitis characterised by (1) acute necrotising granulomas of the upper and lower respiratory tracts, (2) focal necrotizing or granulomatous vasculitis affecting small to medium sized vessels and (3) renal disease in the form of focal or necrotising, often crescentic glomerulitis.
  4. GN - T
  5. Granulomatosis of the upper and lower respiratory tract - T
  6. ANA +ve - F
  7. ANCA +ve - T - c-ANCA is present in 90% of patients with active disease
How well did you know this?
1
Not at all
2
3
4
5
Perfectly
46
Q
  1. Regarding cardiomyopathies, which of the following is the most correct:
    a. Autosomal recessive cases are seen in over 50% of cases of hypertrophic CMP
    b. Heart always increased in weight in dilated CMP
    c. Atria never involved in hypertrophic CMP
    d. Endomyocardial fibrosis is mainly a disease of children and young adults in South America
    e. Dilation of the heart in peripartum CMP is irreversible
A
  1. Heart always increased in weight in dilated CMP - T - characterised by a massively hypertrophied LV without dilatation. Classically, there is disproportionate thickening of the IV septum (asymmetrical septal hypertrophy), although 10% are symmetrical/concentric. LV cavity has a ‘banana-like’ configuration as septum bulges into lumen. Paradoxically provides a markedly reduced stroke volume due to overall small size of chamber and impaired diastolic filling, 25% also have a dynamic LVOT obstruction due to a thickened mitral valve anterior leaflet.
    * RY - this discription is for hypertrophic CM, not dilated. Dilated “usually 2-3x heavier” Big Robbins 9th pg567
  2. Regarding cardiomyopathies, which of the following is the most correct: (GC)
  3. Autosomal recessive cases are seen in over 50% of cases of hypertrophic CMP - F - genetic basis in most cases; 50% familial with autosomal dominant transmission (variable expression). >100 mutations have been identified, the most common are beta-myosin heavy chain, myosin-binding protein C, and troponin T. These 3 genes account for 70-80% of HCM.
  4. Heart always increased in weight in dilated CMP - T - characterised by a massively hypertrophied LV without dilatation. Classically, there is disproportionate thickening of the IV septum (asymmetrical septal hypertrophy), although 10% are symmetrical/concentric. LV cavity has a ‘banana-like’ configuration as septum bulges into lumen. Paradoxically provides a markedly reduced stroke volume due to overall small size of chamber and impaired diastolic filling, 25% also have a dynamic LVOT obstruction due to a thickened mitral valve anterior leaflet.
  5. Atria never involved in hypertrophic CMP - F - left atrial dilatation due to high LV pressures - AF with mural thrombus formation are potential complications. But true if the question refers to LA “hypertrophy” - only LV is hypertrophic.
  6. Endomyocardial fibrosis is mainly a disease of children and young adults in South America - F - technically true but not “most correct”. Primarily a disease of children and young adults in Africa [Robbins]; but is also encountered in tropical and subtropical regions elsewhere in the world, including areas in India and South America that are within 15° of the equator [eMedicine]. EMF is the most common type of restrictive cardiomyopathy worldwide.
  7. Dilation of the heart in peripartum CMP is irreversible in the majority - F - half of these patients will spontaneously recover. Aetiology is multifactorial - preg-assocd HTN, volume overload, nutritional deficiency, metabolic derangement, immunologic response (eg. abnormal cytokine production).
How well did you know this?
1
Not at all
2
3
4
5
Perfectly
47
Q
  1. Aortic dissection: false
    a. Associated with Marfan’s
    b. Stanford classification system
    c. Bicuspid valve
    d. Intimal flap visible in 90%
    e. DeBakey type 2 is the most common
A
  1. DeBakey type 2 is the most common - F - DeBakey Type 1 invovles ascending and descending aorta (50%), Type 2 involves ascending aorta only (10%), Type 3 involves descending aorta (40%)

*LW note small robbins diagram has Debakey images and classification wrong. correct in big robbins.
Remember using pneumonic “BAD”
B: Both ascending and descending aorta - Type 1
A: Ascending: type 2
D: Descending: type 3

  1. Aortic dissection, which is false (JS)
  2. Associated with Marfan’s T - Causes include hypertension, Marfans, Ehlers-Danlos, Aortic coarctation and bicuspid or unicuspid valve
  3. Stanford classification system - T - Stanford classification - Type A involves ascending aorta, Type B involves descending aorta
  4. Bicuspid valve T - Causes include hypertension, Marfans, Ehlers-Danlos, Aortic coarctation and bicuspid or unicuspid valve
  5. Intimal flap visible in 90% - T - Intimal flap is usually present with haematoma spreading between the middle and outer 1/3 of the media.
  6. DeBakey type 2 is the most common - F - DeBakey Type 1 invovles ascending and descending aorta (50%), Type 2 involves ascending aorta only (10%), Type 3 involves descending aorta (40%)
How well did you know this?
1
Not at all
2
3
4
5
Perfectly
48
Q

. Regarding acute rheumatic fever, which of the following is the LEAST CORRECT:

a. It follows pharyngeal infection with group A streptococcus
b. Serofibrinous pericarditis is common
c. Acute arthritis typically affects large joints like the knees
d. Aschoff bodies are an uncommon histological feature of subcutaneous nodules
e. Rheumatic pneumonitis is a recognised rare complication

A
  1. Aschoff bodies are an uncommon histological feature of subcutaneous nodules - F - Aschoff bodies are foci of fibrinoid degeneration surrounded by lymphocytes, plasma cells and macrophages and can be found in any of the three layers of the heart
  2. Regarding acute rheumatic fever, which of the following is the LEAST CORRECT: (JS)
  3. It follows pharyngeal infection with group A streptococcus - T - acute immunologically mediated multisystem inflammatory disease that occurs a few weeks after an episode of group A streptococcal pharyngitis. Major manifestations are migratory polyarthritis, carditis, subcutaneous nodules, erythema marginatum, Sydenham chorea.
  4. Serofibrinous pericarditis is common - T - causes a pancarditis with a serofibrinous pericardial exudate (bread and butter pericarditis), a myocarditis and endocardial involvement
  5. Acute arthritis typically affects large joints like the knees - T - migratory polyarthritis involves the large joints
  6. Aschoff bodies are an uncommon histological feature of subcutaneous nodules - F - Aschoff bodies are foci of fibrinoid degeneration surrounded by lymphocytes, plasma cells and macrophages and can be found in any of the three layers of the heart
  7. Rheumatic pneumonitis is a recognised rare complication T - diffuse pulmonary consolidation and tachypnoea, unresponsive to antibiotic or steriod therapy in a patient with active Rheumatic fever
How well did you know this?
1
Not at all
2
3
4
5
Perfectly
49
Q
  1. Regarding the macroscopic appearance of acute MI, which of the following is the most correct:
    a. The left circumflex coronary shows severe stenosing -
    b. Isolated infarct of the right ventricle occurs in 1-3% of cases -
    c. The median time to rupture is 2-3 days
    d. Pericarditis usually develops at 24hrs post event
    e. Infarcts <48hrs old are usually inapparent on gross examination
A
  1. Isolated infarct of the right ventricle occurs in 1-3% of cases - T
  2. Regarding the macroscopic appearance of acute MI, which of the following is the most correct: (GC)
  3. The left circumflex coronary shows severe stenosing - F - most commonly involves the LAD (40-50%; involves anterior LV, anterior septum, apex), then RCA (30-40%; posterior LV, posterior septum, +/-RV free wall), then left Cx (15-20%; lateral LV except apex).
  4. Isolated infarct of the right ventricle occurs in 1-3% of cases - T
  5. The median time to rupture is 2-3 days - F - can occur at almost any time after MI but is most common at 3-7 days (when lysis of myocardial connective tissue is maximal and granulation tissue has not deposited sufficient collagen matrix to buttress the wall).
  6. Pericarditis usually develops at 24hrs post event - usually develops 2-3 days of transmural MI, resolves with time.
  7. Infarcts <48hrs old are usually inapparent on gross examination - F - at 1-3 days appear mottled with yellow-tan centre.
50
Q
  1. What is not associated with clinical aspect of portal HT:
    a. Ascites
    b. Porto systemic shunts
    c. Congestive splenomegaly
    d. Pancreatitis
    e. Hepatic encephalopathy
A
  1. Pancreatitis
51
Q
  1. Concerning risk factors for atherosclerosis, which of the following is LEAST CORRECT:
    a. Most striking association is with elevated serum levels of LDL
    b. After 45, hypertension is a greater risk factor than hypercholesterolaemia
    c. Smoking increases incidence of sudden death among victims of heart attacks
    d. There is a two fold increase in the incidence of MI in diabetics vs non-diabetics
    e. Systolic hypertension is more important than diastolic hypertension
A

*LW: Robbins and article in journal HTN state: “The risk of cardiovascular disease increases progressively and continuously with increases in SBP and DBP independent of other risk factors; in particular, increased SBP levels are more important than increased DBP”, with Big Robbins stating “ SBP is more important than DBP as a determinent of cardiovascular risk, except in young patients”.
Thus assuming this correlates as risk factor for atherosclerosis, option 5 considered correct.
Therefore, i wonder if incorrect recall, as all are correct.

Previous answer:
5. Systolic hypertension is more important than diastolic hypertension - F - Both systolic and diastolic levels are important in increasing risk

Concerning risk factors for atherosclerosis, which of the following is LEAST CORRECT: (JS)
1. Most striking association is with elevated serum levels of LDL - T

  1. After 45, hypertension is a greater risk factor than hypercholesterolaemia - T:
    * LW: graph of risk factors in Robbins shows a greater increase in risk for increased BP than cholesterol.
  2. Smoking increases incidence of sudden death among victims of heart attacks - ?
    *LW: Couldn’t find this is Robbins. Not stated in Robbins, but generally reported.
    Closest i can find in Robbins “ Smoking increases death rate from IHD by up to 200%”
  3. There is a two fold increase in the incidence of MI in diabetics vs non-diabetics - *LW: T word for word from Robbins.
  4. Systolic hypertension is more important than diastolic hypertension - F - Both systolic and diastolic levels are important in increasing risk *LW disagrees as above.
52
Q
  1. Regarding fatty streaks, which of the following is LEAST CORRECT:
    a. Extracellular lipid is present in smaller amounts than in atheromatous plaques
    b. Streaks decrease in number as atheromatous plaques begin to predominate
    c. 10% of the aortic intimal surface is involved by the third decade of life
    d. Development is independent of racial origin and geography
    e. Distribution of plaques differs from the distribution of fatty streaks.
A

*AJL - I think 5 is false. Robbins says “coronary fatty streaks begin to form in adolescence at the same anatomic sites that later tend to develop plaques suggests a temporal evolution of these lesions.”
I’m unsure of the data related to 3.

**LJS - agree

Previous answers:
3. 10% of the aortic intimal surface is involved by the third decade of life - F - The area of fatty streak develops by 10y and comprises 4-5% of the intimal surface

  1. Regarding fatty streaks, which of the following is LEAST CORRECT: (JS)
  2. Extracellular lipid is present in smaller amounts than in atheromatous plaques T - composed of lipid-filled foam cells with T lymphocytes and extracellular lipid, present in smaller amounts than in plaques
  3. Streaks decrease in number as atheromatous plaques begin to predominate T
  4. 10% of the aortic intimal surface is involved by the third decade of life - F - The area of fatty streak develops by 10y and comprises 4-5% of the intimal surface
  5. Development is independent of racial origin and geography T - Occur in all children over the age of 10y regardless of geography, race, sex or environment.
  6. Distribution of plaques differs from the distribution of fatty streaks. - T - According to Robbins, fatty streaks often occur in areas of vasculature that are not particularly susceptible to developing atheromas later in life.
53
Q
  1. Regarding aortic dissection. Which of the following is the MOST CORRECT:
    a. 5-10% do not have an obvious intimal tear
    b. Hypertension is an antecedent in 50% of cases
    c. Haemorrhage typically occurs between the inner and middle thirds of the media
    d. Elastic fragmentation of the media is rare at autopsy in patients free of dissection
    e. Intimal tear is found in the proximal descending thoracic aorta in 90% of cases
A
  1. 5-10% do not have an obvious intimal tear T - There is usually (but not always) an intimal tear that extends into but not through the media of the ascending aorta (Robbins). According to Primer, intimal flap is detectable in 85-90%.
  2. Regarding aortic dissection. Which of the following is the MOST CORRECT: (JS)
  3. 5-10% do not have an obvious intimal tear T - There is usually (but not always) an intimal tear that extends into but not through the media of the ascending aorta (Robbins). According to Primer, intimal flap is detectable in 85-90%.
  4. Hypertension is an antecedent in 50% of cases F – more than 90% occur in patients with hypertension
  5. Haemorrhage typically occurs between the inner and middle thirds of the media F – haematoma spreads between the middle and outer 1/3 of the media
  6. Elastic fragmentation of the media is rare at autopsy in patients free of dissection - F - cystic medial degeneration (fragmentation and separation of the elastic and fibromuscular elements of the tunica media) is frequently found incidentally at autopsy in patients free of dissection (Robbins)
  7. Intimal tear is found in the proximal descending thoracic aorta in 90% of cases F - typically invovles the ascending aorta
54
Q
  1. Regarding aortic coarctation, which of the following IS NOT a well recognised association:
    a. PDA
    b. Ventriculoseptal defect
    c. Dural AVM
    d. Berry aneurysms of the circle of Willis
    e. Bicuspid valve
A
  1. Dural AVM
  2. Regarding aortic coarctation, which of the following IS NOT a well recognised association: (TW)
  3. PDA – 33%
  4. Ventriculoseptal defect – 15%
  5. Dural AVM
  6. Berry aneurysms of the circle of Willis - associated
  7. Bicuspid aortic valve – assoc with bicuspid aortic valve in 25-50%
    (Dahnert 6th ed.) Coarctation – 50% associated with
    o bicuspid arotic valve (in 25-50%)
    o intracardiac malformations – PDA, VSD, AS, AR, ASD, TGV, ostium primum defect, truncus arteriosus, double-outlet RV.
    o Non-cardiac malformations. Turner syndrome.
    o Cerebral berry aneurysms
    o Mycotic aneurysm distal to CoA.
55
Q
  1. Regarding pulmonary emboli and infarcts, which of the following is MOST CORRECT:
    a. Majority (>50%) result in white infarcts because it is an end circulation
    b. 75% are in the lower lobes
    c. Pulmonary haemorrhage suggest infarction
    d. The presence of viable alveoli pneumocytes/endothelium in an area of haemorrhage is against a diagnosis of PE
    e. >50% of infarcts are solitary
A
  1. 75% are in the lower lobes -T
  2. Regarding pulmonary emboli and infarcts, which of the following is MOST CORRECT: (JS)
  3. Majority (>50%) result in white infarcts because it is an end circulation - F - Classically haemorrhagic
  4. 75% are in the lower lobes -T
  5. Pulmonary haemorrhage suggest infarction - F - the bronchial arteries can sustain the lung parenchyma despite obstruction of pulmonary arterial system and under these circumstances, haemorrhages may occur with no infarction. Only about 10% of emboli cause infarction (typically when circulation is inadequate such as heart or lung disease). (Robbins)
  6. The presence of viable alveoli pneumocytes/endothelium in an area of haemorrhage is against a diagnosis of PE - F - haemorrhages (I presume related to pe) are distinguished by preservation of pulmonary architecture
  7. > 50% of infarcts are solitary - F - in more than half of cases, multiple lesions occur
56
Q
  1. Regarding infective endocarditis, which of the following is the LEAST CORRECT:
    a. Right sided heart valves in 50% of narcotic related cases
    b. Diffuse glomerulonephritis seen in 50% of untreated cases
    c. Positive blood cultures are seen in 85-90% of cases
    d. Strep pneumonia is the leading cause of acute endocarditis
    e. With mechanical prostheses infections are usually located on the margin of the sewing ring
A
  1. Strep pneumonia is the leading cause of acute endocarditis – F – (note that path notes say more than ½ cases due to various streptococci – however most commonly gingival disease) – since 1980s IE due to S aureus (primary pathogen) primary risk factor is intravascular devices (eMed).
  2. Regarding infective endocarditis, which of the following is the LEAST CORRECT: (TW)
  3. Right sided heart valves in 50% of narcotic related cases – T - in 75% of IVDA IE, no underlying valvular abnormalities are noted, and 50% of these infections involve the tricuspid valve (eMed).
  4. Diffuse glomerulonephritis in 50% of untreated cases – T ~30% UpToDate
  5. Positive blood cultures in 85-90% of cases – T – path notes / UpToDate (but varies depending on series and geography)/ eMed
  6. Strep pneumonia is the leading cause of acute endocarditis – F – (note that path notes say more than ½ cases due to various streptococci – however most commonly gingival disease) – since 1980s IE due to S aureus (primary pathogen) primary risk factor is intravascular devices (eMed).
  7. With mechanical prostheses infections are usually located on the margin of the sewing ring – T – endocarditis is located at the prosthesis-tissue intervace, causing a ring intervace (Robbins).
57
Q

September 200417. Hypertensive heart disease, which is unlikely:
1. Ventricular hypertrophy and mild dilatation

A

???

  • AJL
  • Left sided (systemic) HTN results in left ventricular thickening (early) and eventually restriction and atrial dilation.
  • Right sided (pulmonary, cor pulmonale) HTN results in right ventricular thickening and dilation.
58
Q
  1. Aortic dissection: (presumably which is false)
  2. Can occur without an intimal flap
  3. Longitudinal
  4. 30-50cm long
  5. Aortic valve prevents involvement coronary arteries
A
  1. Longitudinal - F - tears are usually transverse or oblique, 1-5cm in length with sharp jagged edges
  2. Aortic valve prevents involvement coronary arteries - F - occlusion of coronary aa in 8%
  3. Aortic dissection: (presumably which is false) (JS)
  4. Can occur without an intimal flap - T - There is usually (but not always) an intimal tear that extends into but not through the media of the ascending aorta (Robbins). According to Primer, intimal flap is detectable in 85-90%.
  5. Longitudinal - F - tears are usually transverse or oblique, 1-5cm in length with sharp jagged edges
  6. 30-50cm long ? - not sure if they mean the length of the tear (1-5cm) or the length of the dissection which is variable
  7. Aortic valve prevents involvement coronary arteries - F - occlusion of coronary aa in 8%
59
Q
  1. Regarding cardiomyopathies, which of the following is the most correct:
  2. Can involve the right ventricle
  3. Autosomal recessive cases are seen in over 50% of cases of hypertrophic CMP
  4. Atria never involved in hypertrophic CMP
  5. Endomyocardial fibrosis is mainly a disease of children and young adults in South America
  6. Dilation of the heart in peripartum CMP is irreversible in the majority
A
  1. Can involve the right ventricle - T - dilation seen in all 4 chambers in DCM
  2. Regarding cardiomyopathies, which of the following is the most correct: (GC)
  3. Can involve the right ventricle - T - dilation seen in all 4 chambers in DCM
  4. Autosomal recessive cases are seen in over 50% of cases of hypertrophic CMP - F - genetic basis in most cases; 50% familial with autosomal dominant transmission (variable expression). >100 mutations have been identified, the most common are beta-myosin heavy chain, myosin-binding protein C, and troponin T. These 3 genes account for 70-80% of HCM.
  5. Atria never involved in hypertrophic CMP - F - left atrial dilatation due to high LV pressures - AF with mural thrombus formation are potential complications. But true if the question refers to LA “hypertrophy” - only LV is hypertrophic.
  6. Endomyocardial fibrosis is mainly a disease of children and young adults in South America - F - technically true but not “most correct”. Primarily a disease of children and young adults in Africa [Robbins]; but is also encountered in tropical and subtropical regions elsewhere in the world, including areas in India and South America that are within 15° of the equator [eMedicine]. EMF is the most common type of restrictive cardiomyopathy worldwide.
  7. Dilation of the heart in peripartum CMP is irreversible - F - half of these patients will spontaneously recover. Aetiology is multifactorial - preg-assocd HTN, volume overload, nutritional deficiency, metabolic derangement, immunologic response (eg. abnormal cytokine production).
60
Q
  1. Wegners, which is against the diagnosis:
  2. involvement of nasal mucosa
  3. Renal involvement
  4. c ANCA
  5. Female
  6. Necrotizing vasculitis
  7. Necrotizing glomerular nephritis
  8. Neg ANCA
  9. Granulomatosis of upper and lower resp tracts
A
  1. Neg ANCA - F - c-ANCA is present in 90% of patients with active disease
  2. Wegeners, which is against the diagnosis: (JS)
  3. involvement of nasal mucosa
  4. Renal involvement
  5. c ANCA
  6. Female - ?? - Men are affected more often than women (2:1), at an average age of 40y. But if you have other features of the disease, I don’t think being female would be against the diagnosis…
  7. Necrotizing vasculitis
  8. Necrotizing glomerular nephritis
  9. Neg ANCA - F - c-ANCA is present in 90% of patients with active disease
  10. Granulomatosis of upper and lower resp tracts

Wegener Granulomatosis : A necrotising vasculitis characterised by (1) acute necrotising granulomas of the upper and lower respiratory tracts, (2) focal necrotizing or granulomatous vasculitis affecting small to medium sized vessels and (3) renal disease in the form of focal or necrotising, often crescentic glomerulitis.

61
Q
  1. Elderly man with polycythaemia. Non-associated finding on CT is:
  2. Emphysema
  3. Renal mass
  4. Chronic pancreatitis
  5. Splenomegaly
  6. HCC
A
  1. Chronic pancreatitis - F
  2. Elderly man with polycythaemia. Non-associated finding on CT is: (TW)
  3. Emphysema - T - hypoxiemia secondary to chronic pulmonary disease, right-to-left shutns, sleep apnoea, massive obesity (Pickwickian syndrome), high altitude.
  4. Renal mass - T - EPO producing neoplasms - RCC, HCC, hemangioblastoma, uterine fibroids.
  5. Chronic pancreatitis - F
  6. Splenomegaly - T - polycythema vera / polycythemia may have splenomegaly. The presence of hepatomegaly and splenomegaly in a patient with polycytmeia may reflect the presence of polycythemia vera or an EPO-secreting hepatoma.
  7. HCC - T - see ans 2.
    Inappropriately high serum EPO - autonomous (inappropriate) increase of EPO: EPO-producing neoplasms. EPO-produing renal lesions.
    Appropriately high serum EPO - appropriate increases in EPO: hypoxemia
    Germline and somatic mutation causes of polycythemia: eg polycythemia rubra vera, mutations of EPO receptor, methemaglobinemia, idiopathic familial.
62
Q
  1. Pt with mononucleosis, which is atypical?:
  2. Urinary epithelial with inclusions
  3. Atypical lymphocytes
A
  1. Urinary epithelial with inclusions - F - this refers to polyoma viruses (BK and JC). A morphologic sign of the (re)activation of polyomaviruses is the detection of typical intranuclear viral inclusion bearing cells, so-called “decoy-cells”, in the urine. But a number of things can cause inclusions.
  2. Pt with mononucleosis, which is atypical?: (TW)
  3. Urinary epithelial with inclusions - F - this refers to polyoma viruses (BK and JC). A morphologic sign of the (re)activation of polyomaviruses is the detection of typical intranuclear viral inclusion bearing cells, so-called “decoy-cells”, in the urine. But a number of things can cause inclusions.
  4. Atypical lymphocytes - T - most common finding in association with infectious mononucleosis is lymphocytosis. Smear may also identify significant atypical lymphocytosis, defined as more than 10% of total lymphocytes.
63
Q
  1. Mitral ring calcification:
  2. Soft
  3. Hard, MVR & conduction defects
  4. Normal variant
A
  1. Hard, MVR & conduction defects

Irregular, stony hard that lie behind the leaflets. It usually does not affect valvular function but in unusual cases can cause stenosis and arrhythmia secondary to impingement on conduction system. Most common in women > 60 years.

64
Q
  1. 40 yr old asthmatic, autopsy. 2 valves, one with a septum, ? true:
  2. Congenital bicuspid valve
  3. Rheumatic heart disease
A
  1. Congenital bicuspid valve
65
Q
  1. ASD secundum, ? true:
  2. 60% of ASD’s
  3. Usually isolated
A
  1. Usually isolated - T - Robbins says “most are isolated (not associated with other anomalies). UTD says: secundum ASDs may be associated with other ASDs, such as a sinus venosus defect or an ostium primum defect. ASDs are often associated with other congenital cardiac anomalies.
    * LW: Robbins states 90% of ASDs = secundum, are usually without other cardiac abnormalities, hence answer 2 correct.

(Primum ASD, less common at 5%, are often associated with other cardiac abnormalities originating from endocardial cushion).

  1. ASD secundum, ? true: (TW)
  2. 60% of ASD’s - F - path notes say 90%. UTD says secundum defects account for approx 70% of all ASDs and occur twise as often in females as in males (UTD)
  3. Usually isolated - T - Robbins says “most are isolated (not associated with other anomalies). UTD says: secundum ASDs may be associated with other ASDs, such as a sinus venosus defect or an ostium primum defect. ASDs are often associated with other congenital cardiac anomalies.
66
Q
  1. PE and infarcts, which is true:
  2. 75% lower zones
  3. Pulm hemorrhage suggests infact
  4. > 50% white infarcts b/c of endarterial circulation
  5. Presence of viable alveoli, endothelium in an area of hemorrhage is against diagnosis of PE
A
  1. 75% lower zones T - 75% affect lower lobes (path notes). Dahnert says (about) 25% each LL, about 15% for ULs and ML.
  2. PE and infarcts, which is true: (TW)
  3. 75% lower zones T - 75% affect lower lobes (path notes). Dahnert says (about) 25% each LL, about 15% for ULs and ML.
  4. Pulm hemorrhage suggests infact - F - 25-34% present with hemoptysis. Only 10-60% of PE’s have infarction. Up to 90% do not have infarction associated, but histologically shows hemorrhage and oedema (D). Collateral circulation of lung - dual pulmonary blood supply and only pulmonary arteries involved in PE. Haemorrhage results without lung infarction. Inadequate bronchial artery supply usually results in lung infarct (path notes).
  5. > 50% white infarcts b/c of endarterial circulation - F - 10-15% small arteries “end arteries” - infarction (path notes).
  6. Presence of viable alveoli, endothelium in an area of hemorrhage is against diagnosis of PE - - F - Pulmonary haemorrhage is much commoner than infarction. Underlying pulmonary architecture is preserved, suffused initially, resportion usual, +/- fibrinous pleuritis.
67
Q
  1. Young female with tufts of capillary formation?
  2. Primary pulmonary hypertension
  3. Goodpastures
  4. SLE
  5. Recurrent thromboemboli
A
  1. Primary pulmonary hypertension - T - Primary (or idiopathic) pulmonary hypertension is uncommon, typically occurring in women 20-40yo. Vascular lesions include atheromas in large elastic arteries, intimal fibrosis or medial hypertrophy in medium-sized muscular arteries and smaller arterioles. So-called ‘plexogenic arteriopathy’ (tufts within capillary channels creating a vascular plexus), seen in primary pulmonary hypertension and with some congenital cardiovascular anomalies.
68
Q
  1. Mycotic aneurysms, which is false?
  2. Rarely bleed
  3. Peripheral arterial
  4. Circle of Willis
  5. Salmonella accounts for over 1/3
  6. Alcoholism predisposes
A
  1. Rarely bleed - F - life-threatening rupture and haemorrhae in 75%. If specific to CNS, then also true - MAs develop recurrent bleeding more frequently than congenital aneurysms.
  2. Mycotic aneurysms, which is false? (GC)
  3. Rarely bleed - F - life-threatening rupture and haemorrhae in 75%. If specific to CNS, then also true - MAs develop recurrent bleeding more frequently than congenital aneurysms.
  4. Peripheral arterial - T - sites: ascending aorta near sinus of Valsalva > abdominal visceral a. > intracranial a. > lower or upper extremity a.
    If specific to CNS mycotic A, then also true - cerebral MAs tend to occur in the more distal portions of the MCA near the surface of the brain involving the secondary and tertiary branches.
  5. Circle of Willis - T - peripheral to the first bifurcation of a major vessel, may occur in suprasellar cistern, ie. COW. [Dahnert]. However, berry aneurysms more typically occur in or near the COW. The MCA territory is involved 4 times more commonly with mycotic aneurysms than either the anterior or posterior cerebral arteries. [eMedicine]
  6. Salmonella accounts for over 1/3 - T - 3 S’s : Staph (53%), Salmonella (33-50%), nonhaemolytic Strep. Also pneumococcus, gonococcus, contiguous spread from mycobacterium in spine/LNs.
  7. Alcoholism predisposes - T - immunocompromised status (also malignancy, chemotherapy, steroids, AI disease, diabetes), IVDU, bacterial endocarditis (12%), atherosclerosis, aortic trauma (accidents, iatrogenic).
69
Q
  1. 60 year old with myeloma and restrictive heart disease, ? cause:
  2. AL
  3. AA
A
  1. AL - T - AL (primary) amyloid can occur alone or in association with multiple myeloma, or much less often, Waldenstrom’s macroglobulinemia.
  2. 60 year old with myeloma and restrictive heart disease, ? cause: (TW)
  3. AL - T - AL (primary) amyloid can occur alone or in association with multiple myeloma, or much less often, Waldenstrom’s macroglobulinemia.
  4. AA - F - AA (secondary) amyloidosis may complicate chronic diseases in which there is ongoing or recurring inflammation (eg RA, or spondyloarthropathy; chronic infections; or periodic fever syndromes).
    Amyloidosis refers to the extracellular tissue deposition of fibrils that are composed fo low molecular weight subunits of a variety of serum proteins. These fibrils adopt a beta-pleated sheet configuration that lead to the characteristic histologic changes.
    Fragments of serum amyloid A protein, an acute phase reactant, are responsible for AA (secondary) amyloidosis, which is associated with a variety of chronic inflammatory disorders. However, AA amyloidosis almost never produces clinically apparent heart disease.
70
Q
  1. Causes of pericarditis – least likely:
  2. TB
  3. chronic renal failure
  4. cocksackie virus
  5. post MI
  6. Cryptococcus
A
  1. Cryptococcus - F
    Generally, malignancy, infection and idiopathic are the most common causes (but depends on which series you look at and which country)
  2. Causes of pericarditis – least likely: (GC, TW)
  3. TB - T - tuberculous pericarditis is invariable associated with TB elsewhere in the body. Occurs in 1-2% of patients with pulmonary TB.
  4. chronic renal failure - T - uraemic pericarditis is observed in 6-10% of patients with advanced renal failure (acute or chronic) before dialysis has been instituted or shortly thereafter. Peridarditis also occursi n approx 13% of patients on maintenance hemodialysis, and may be seen with chronic peritoneal dialysis.
  5. cocksackie virus - T - the most common viral infections causing pericarditis include coxsackievirus A and B, echovirus, adenovirus, and HIV.
  6. post MI - T - Dressler’s syndrome
  7. Cryptococcus - F
    Generally, malignancy, infection and idiopathic are the most common causes (but depends on which series you look at and which country)
    Dahnert: CUM TAPPIT RV
    Collagen vasc disease, Uraemia, Mets, Trauma, AMI , Post MI, Purulent infection, Idiopathic, TB, RA/RHD, Virus.
71
Q
  1. Type B dissection - definition:
  2. Involves asc and desc aorta
  3. Arises distal to left SCA
  4. Arises distal to left CCA
  5. Arises distal to ligamentum arteriosum
  6. Involves descending thoracic aorta
A

*LW:
Preferred answer is 3: arises distal to left SCA.

“By definition, type A dissection involves the ascending aorta, whereas type B dissection is limited to the aorta distal to the left subclavian artery” RG article.

Type B: B begins beyond brachiocephalic vessels
accounts for ~40% of aortic dissections.
Dissection commences distal to the left subclavian artery
medical management with blood pressure control. Radiopedia.

Old answer:
5. Involves desc thoracic aorta

Stanford B – begins beyond brachiocephalic vessels. Stanford A affects ascending aorta and arch. Stanford B is confined to the descending aorta (DeBakey III) – descending aorta starts at aortic isthmus (area between left SCA and ligamentum arteriosus).

72
Q
  1. Suspected bacterial endocarditis - which is least likely?
  2. Candida
  3. repeated negative blood cultures
  4. liver infarct
  5. hematuria
  6. MCA aneurysm
A
  1. liver infarct - F - liver rarely infarcts as 70% blood is derived from PV.
  2. Suspected bacterial endocarditis - which is least likely? (GC)
  3. Candida - T - fungi, rickettsiae (Q fever), and chlamydiae may cause IE but far less common than extracellular bacteria. Strep viridans (50-60%), staph (10-20%, esp. IVDU), HACEK group of oral commensals (haemophilus, actinobacillus, cardiobacterium, eikenella, kingella).
  4. repeated negative blood cultures - T - blood cultures +ve in 80-95% (10% are “culture-negative”).
  5. liver infarct - F - liver rarely infarcts as 70% blood is derived from PV.
  6. hematuria - T - glomerulonephritis due to Ag-Ab complexes may give rise to haematuria, albuminuria, or renal failure.
  7. MCA aneurysm - T - subacute bacterial endocarditis, IVDU; occur more peripherally and often multiple.
    [Robbins]
73
Q
  1. An article on myocardial infarction talks of “reperfusion injury”. This is best described by?
  2. A special form of damage, probably free radical mediated
  3. The injury that persists after revascularisation of an infarct
  4. Haemorrhage into an area of ischemic necrosis
  5. Resolution of early reversible ischemic cellular changes
  6. Myocardium viable but shows a transitory depression of function
A
  1. A special form of damage, probably free radical mediated - reperfusion injury can incite greater local damage than might have otherwise occurred without rapid restoration of flow; mediated in part by oxygen free radicals generated by the increased no. of infiltrating leukocytes facilitated by reperfusion. Reperfusion-induced microvascular injury causes not only haemorrhage but also endothelial swelling that occludes capillaries and may prevent local blood flow (called “no-reflow”).
  2. An article on myocardial infarction talks of “reperfusion injury”. This is best described by? (GC)
  3. A special form of damage, probably free radical mediated - reperfusion injury can incite greater local damage than might have otherwise occurred without rapid restoration of flow; mediated in part by oxygen free radicals generated by the increased no. of infiltrating leukocytes facilitated by reperfusion. Reperfusion-induced microvascular injury causes not only haemorrhage but also endothelial swelling that occludes capillaries and may prevent local blood flow (called “no-reflow”).
  4. The injury that persists after revascularisation of an infarct - F
  5. Haemorrhage into an area of ischemic necrosis - F
  6. Resolution of early reversible ischemic cellular changes - F
  7. Myocardium viable but shows a transitory depression of function - F - “stunned” myocardium; due to abnormalities in cellular biochemistry that persist for several days after ischaemia and lead to noncontractile state.
    [Robbins]
74
Q
  1. Concerning fat embolism, which of the following statements is least correct?
  2. Over 80% of fat emboli are asymptomatic
  3. Features of fat embolism include hypoxia, thrombocytopenia, and CNS symptoms
  4. symptoms are usually established within 6 hours of trauma/fracture
  5. neurological manifestations include irritability, restlessness, confusion, and coma
  6. 20-50% of cases are associated with purpuric rash
A
  1. symptoms are usually established within 6 hours of trauma/fracture - F - typically 1-3 days after injury
  2. Concerning fat embolism, which of the following statements is least correct? (GC)
  3. Over 80% of fat emboli are asymptomatic - T - symptomatic fat embolism syndrome in <10%.
  4. Features of fat embolism include hypoxia, thrombocytopenia, and CNS symptoms - T - systemic hypoxaemia and dyspnoea, mentation changes (headaches, confusion), petechiae in 50% from coagulopathy (release of tissue thromboplastin), fever.
  5. symptoms are usually established within 6 hours of trauma/fracture - F - typically 1-3 days after injury
  6. neurological manifestations include irritability, restlessness, confusion, and coma - T - see ans (2).
  7. 20-50% of cases are associated with purpuric rash - T - see ans (2).
    [Dahnert]
75
Q
  1. Woman has an irregularity of the aortic arch for which past dissection, Takayasu arteritis, and giant cell arteritis are considered. Which of the following statements is most correct?
  2. If the patient is less than 40 yo, past dissection should be discounted
  3. If the patient is female, Takayasu arteritis would be much less likely
  4. If the patient is over 70 yo Takayasu would be strongly favoured
  5. If there is renal vessel narrowing, past dissection would be strongly favoured
  6. If there are giant cells in the media and patient is > 50 yo, Takayasu disease is less likely, and giant cell more likely
A

e. If over 50yrs and giant cells on biopsy then giant cell arteritis should be considered. – T - affects older people (rare <50yo). Giant cells present in only 2/3rds. Most common systemic vasculitis in adults.

  1. Woman has an irregularity of the aortic arch for which past dissection, Takayasu arteritis, and giant cell arteritis are considered. Which of the following statements is most correct? (TW)
    a. If patient is < 40 yrs old, past dissection should be discounted – F – although most are 40-60yo men with HT, can have connective tissue disorders (eg Marfans), or post arterial cannulation, during pregnancy (path notes)
    b. If the patient is female , Takayasu arteritis would be less likely – F – would be more likely. TA affects females 15-40yo, most common in Asia.
    c. If the patient is over 70 yrs Takayasu arteritis is strongly favoured - F – less favoured. See ans to (a)
    d. If there is renal vessel narrowing, past dissection would be strongly favoured. – F - Takayasu’s arteritis classically involves arch and ostis of major branchs, however 1/3 also affect the remainder of the aorta and its branches. Some cases are limited to descending thoracic and abdominal aorta.
    e. If over 50yrs and giant cells on biopsy then giant cell arteritis should be considered. – T - affects older people (rare <50yo). Giant cells present in only 2/3rds. Most common systemic vasculitis in adults.
76
Q
  1. Concerning PE, which of the following is most correct?
  2. Approximately 40% lead to infarction
  3. Emboli affect upper, mid and lower zones equally but not lobes equally
  4. In the young, PE more frequently leads to infarction
  5. In the presence of a predisposing cause, secondary episodes may occur up to 30%
  6. Pulmonary haemorrhage in PE implies reperfusion of an infarct
A

d. In the presence of a predisposing cause, secondary episodes may occur in up to 30% - T – 30% develop a second embolus (path notes).

  1. Concerning PE, which of the following is most correct? (TW)
    a. 40% lead to infarction – F – dual blood supply. Embolism without infarction 90%, embolism with infarction 10-60% (D 6th ed.). Path notes say 10% only.
    b. Emboli affect upper, mid and lower lobe zones equally – F – bilat 45%, right lung only 36%, left lung only 18%. (LL > UL > ML) LLL 26%, RLL 25%, RUL 16%, LUL 14%, RML 9%.
    c. In the young PE more frequently leads to infarction – F - see increased pulmonary infarction if abnormal bronchial supply, usually assoc with underlying heart – lung disease. Would be less in younger groups.
    d. In the presence of a predisposing cause, secondary episodes may occur in up to 30% - T – 30% develop a second embolus (path notes).
    e. Pulmonary haemorrhage implies reperfusion of an infarct. – F – pulmonary haemorrhage is much commoner than infarction (path notes). In patients with adequate CV function, the bronchial artery supply can often sustain the lung parenchyma despite obstruction to the pulmonary arterial system Under these circumstances, haemorrhages may occur, but there is no infarction (Robbins)
77
Q
  1. Most at risk for cerebral venous infarction
  2. Diabetic in renal failure
  3. Post obstetric patient
  4. Young girl on progesterone only pill
  5. Young girl with SLE
A
  1. Post obstetric patient - T -
  2. Most at risk for cerebral venous infarction: (GC)
  3. Diabetic in renal failure - F
  4. Post obstetric patient - T -
  5. Young girl on progesterone only pill - F
  6. Young girl with SLE - F

> 100 causes of CVT have been described.
Local - related to intrinsic or mechanical conditions of the cerebral vv or dural sinuses; eg. sinus trauma, mastoiditis, neoplastic.
Systemic - related to hypercoagulable states; eg. protein S and protein C deficienceies, peripartum, OCP use, malignancy.
[RG 2006]

(–) International Study on Cerebral Vein Thrombosis (ISCVT) is a multinational, multicenter prospective observational study whose objectives are to describe the long-term prognosis of CVT, to identify predictors of outcome, and to describe variability of CVT management and the associated risk factors and conditions. Regarding Venous infarction, the most frequent identified predisposing conditions were: oral contraceptives (46%), prothrombotic conditions (27%), pregnancy/puerperium (17%), ear-nose-throat infections (8%), central nervous system (CNS) infections (5%), other infections (4%), CNS disorders (5%), cancer (8%), vasculitis/antiphospholipid antibodies (8%), and other systemic diseases (14%). The cause was unknown in only 14% of patients.

78
Q
  1. A patient has prominent pulmonary artery, which of the following histories is least likely cause of pulmonary arterial hypertension?
  2. 38 yo smoker with basal emphysema
  3. 32 yo female whose mother died of primary pulmonary arterial hypertension
  4. 2 yo refugee with pansystolic machinery type cardiac murmur
  5. 30 yo female diabetic with chronic renal failure
  6. 22 yo with SLE with recurrent right leg swelling
A
  1. 30 yo female diabetic with chronic renal failure
  2. A patient has prominent pulmonary artery, which of the following histories is least likely cause of pulmonary arterial hypertension? (–)
  3. 38 yo smoker with basal emphysema - Alpha 1 AT (True)
  4. 32 yo female whose mother died of primary pulmonary arterial hypertension – some familial transmission
  5. 2 yo refugee with pansystolic machinery type cardiac murmur - PDA
    * *LJS - seems it is possible to get pulmonary vascular changes by 2 years of age due to PDA - https://onlinelibrary.wiley.com/doi/full/10.1111/chd.12702
  6. 30 yo female diabetic with chronic renal failure
    * *LJS - CRF is also a cause of PAH
  7. 22 yo with SLE with recurrent right leg swelling - recurrent PE
79
Q
  1. Cardiac echo with focal abnormal ventricular wall motion - least likely cause
  2. AMI
  3. Old infarct
  4. Stunned myocardium
  5. Hibernating myocardium
  6. Endomyocardial fibroelastosis
A
  1. Endomyocardial fibroelastosis - can cause focal but rare disease
  2. Cardiac echo with focal abnormal ventricular wall motion - least likely cause (TW)
  3. AMI T
  4. Old infarct T
  5. Stunned myocardium - T - stunned myocardium follows an episode of acute coronary ischaemia (cf hibernating myocardium which occur sin clinical syndromes of unstable and chronic stable angina, acute MI, and LV dysfunction or heart failure). Time course of recovery can be <2h to >24hr depending on the degree of ischameia.
  6. Hibernating myocardium - ‘Hibernating myocardium is a state of persistently impaired myocardial impaired myocardial and LV function at rest due to chronically reduced coronary blood flow that can be partially or completely restored to normal either by improving blood flow or by reducing oxygen demand.
  7. Endomyocardial fibroelastosis - can cause focal but rare disease
    Stem c is a tricky one as stunned myocardium does show reduced motion but only temporarily. SO unless recent MI, C is likely false.
80
Q
  1. Which is the least concern for bleeding post biopsy:
  2. Female with maternal fam history of haemophilia
  3. Von Willebrands with normal APTT (?PT)
  4. Haemophilia with normal platelets
  5. Widespread prostate mets
A
  1. Female with maternal fam history of haemophilia - T - hemophilia A and B are X-linked recessive disorders. These hemophilias occur almost excluslvely in a male having one defective copy of the relevant gene on his X-chromosome. Transmission from father will only be to daughters (carriers - heterogozygous), and sons will not be affected. Although female carriers have one normal allele, they may experience bleeding symptoms similar to that seen in a patient with ‘mild’ deficiency. Some women however may have more symptomatic hemophilia.
  2. Which is the least concern for bleeding post biopsy: (TW)
  3. Female with maternal fam history of haemophilia - T - hemophilia A and B are X-linked recessive disorders. These hemophilias occur almost excluslvely in a male having one defective copy of the relevant gene on his X-chromosome. Transmission from father will only be to daughters (carriers - heterogozygous), and sons will not be affected. Although female carriers have one normal allele, they may experience bleeding symptoms similar to that seen in a patient with ‘mild’ deficiency. Some women however may have more symptomatic hemophilia.
  4. Von Willebrands with normal APTT (?PT) - F - if the apTT is prolonged and the PT (INR) is normal, the problem is localized to the intrinsic pathway. The most common inherited bleeding disorders giving this picture are VW disease, and isolated deficiencies of factors VIII, IX, and XI. If both aPTT and the PT (INR) are normal in a patient with an apparent bleeding diathesis, then thrombocytopenia, mild deficiency of vWf, platelet dysfunction, etc should be considered.
  5. Haemophilia with normal platelets
  6. Widespread prostate mets
81
Q
  1. Chronic otits media / mastoiditis, renal lesion, hilar mass. FNA shows non specific granuloma, best explained by:
  2. Wegner granulomatosis
  3. Disseminated TB
  4. Disseminated sarcoid
  5. Polyarteritis nodosa
  6. Goodpastures disease
A
  1. Wegner granulomatosis Chronic suppurative otitis media is present in 24% of patients with WG. …
82
Q
  1. Patient with previous Goodpastures, on dialysis. Gross hematuria and hypotensive. ?cause
  2. Prostate vol 25cc
  3. Renal cysts
  4. Renal calculi
  5. Amyloid
A

*LW:
I would favour renal cysts as correct answer, as dialysis patients commonly get renal cysts which can be come large, and are at risk for rupture, which can bleed accounting for hypotension and haematuria (good old clinical renal medicine)

  1. Amyloid - T - GPS = anti-GBM-Ab crescenteric GN, aka type I CrGN or rapidly progressive GN (linear deposits of IgG +/- C3).
83
Q
  1. Hypertensive patient most likely findings:
  2. 2cm adrenal Ca
  3. 10cm adenoma
  4. Bilateral benign tumours
  5. Bilateral hyperplasia
  6. Adrenal atrophy secondary to sustained HTN
A
  1. Bilateral hyperplasia Nonspecific hyperplasia is associated with 15% of hypertensives (Primer). Responsible for 8% of Cushing syndrome and 10-20% hyperaldosteronism. Incidence of cushings syndrome – 1:1000. 70% of these is from Cushing disease (adrenal hyperplasia due to overproduction of pituitary ACTH).
  2. Hypertensive patient most likely findings: (TW)
  3. 2cm adrenal Ca F – 1:1000000. Usually >5cm, median size 12cm; in 16% <6cm. 50% hyperfunctioning. 20% non-functioning.
  4. 10cm adenoma F – usually <5cm (avg 2-2.5cm). Aldosteronoma usually small 1.7cm avg.
  5. Bilateral benign tumours ?Bilateral adrenal indicentalomas found in 10-15% of cases. Majority of adrenal indicentalomas are non functional (15% functional). Of total – 85% non functioning, 9% subclinical Cushings, 4% phaeos, 2% aldosteronomas (UpToDate).
  6. Bilateral hyperplasia Nonspecific hyperplasia is associated with 15% of hypertensives (Primer). Responsible for 8% of Cushing syndrome and 10-20% hyperaldosteronism. Incidence of cushings syndrome – 1:1000. 70% of these is from Cushing disease (adrenal hyperplasia due to overproduction of pituitary ACTH).

Primer – Nonspecific hyperplasia is also associated with: Acromegaly 100%, Hyperthyroidism 40%, Hypertension 15%.
Est 30% of >18yo have hypertension and increases with age – 50% age 60yo (in USA, UpToDate). Most of these (90%) are essential HTN.
Incidental adrenal masses found in 9% normotensive patients, and 12% of hypertensive patients (90% of these 12% are essential HTN). From numbers in option ‘c’, ~1% of the essential HTN’s have bilateral adrenal incidentalomas. Where as 15% of hypertensives have nonspecific bilateral adrenal hyperplasia.

84
Q
  1. Vascular 5cm renal mass found on investigation for RAS which was normal. ?Most likely
  2. incidental RCC
  3. RCC as cause for hypertension
  4. Oncocytoma
  5. Need to check for urinary VMA before biopsy
A
  1. RCC as cause for hypertension (RCC causes a variety of paraneoplastic syndromes includeing polycythaemia,hypercalciaemia,heypetension, cushings syndrome etc)
  2. Vascular 5cm renal mass found on investigation for RAS which was normal. ?Most likely (–)
  3. incidental RCC:
    * LW: I would consider this option could be correct also, as “95% of HTN is idiopathic, thus probability is that patient has essential HTN and a RCC, as not all RCC are paraneoplastic (only 1/3), and not all paraneoplastic RCC’s result in HTN.

*WJI: disagree with logic. We are not trying to work out if a person with HTN has a RCC (as above, >95% chance they don’t).
RCCs are 2% of all cancers, not 100% of people have cancer so risk of incidental RCC is significantly less than 2%
20% of people with RCC have paraneoplastic HTN (radiopaedia).
20% is more than 2% so 2. is more correct than 1.

  1. RCC as cause for hypertension (RCC causes a variety of paraneoplastic syndromes includeing polycythaemia,hypercalciaemia,heypetension, cushings syndrome etc)
  2. Oncocytoma
  3. Need to check for urinary VMA before biopsy (pheao not renal)
85
Q
  1. Angiodysplasia -?Most likely
  2. Most commonly in sigmoid
  3. Can’t be seen macroscopically because covered by mucosa
  4. Cause of 20% significant lower GI bleeding – either as massive blood loss or chronic loss
  5. Not assoc with other GIT lesions or CVS abnormalities
A
  1. Cause of 20% significant lower GI bleeding – either as massive blood loss or chronic loss
  2. Angiodysplasia -?Most likely (–)
  3. Most commonly in sigmoid F - caecum
  4. Can’t be seen macroscopically because covered by mucosa F
  5. Cause of 20% significant lower GI bleeding – either as massive blood loss or chronic loss (direct quote from robbins)
  6. Not assoc with other GIT lesions or CVS abnormalities (angiiodysplasia associated with meckels and aortic stenosis)
86
Q
  1. Question on giant cell arteritis:
  2. only in the ECA
  3. lesions in continuity
  4. Multinucleated giant cells and Langhans cells
  5. Only temporal artery
A
  1. Multinucleated giant cells and Langhans cells - T - MNGC, both foreign body and Langerhans cells, present in 2/3.
  2. Giant cell arteritis: (GC)
  3. only in the ECA - F - principally affects the arteries of head (esp. the temporal a.) but can also involve vertebral and ophthalmic arteries. Giant cell aortitis may also occur.
  4. lesions in continuity - F - lesions are not continuous along the vessel, and long segments of relatively normal artery may separate areas of inflammation.
  5. Multinucleated giant cells and Langhans cells - T - MNGC, both foreign body and Langerhans cells, present in 2/3.
  6. Only temporal artery - F - extraordinary predilection for a single vascular site (temporal a.), but other arteries can be involved (see ans 1).
    [Robbins]
87
Q
  1. 8yo Downs syndrome child - Becomes anemic + splenomegaly + thrombocytopenia. Likely cause:
  2. Right heart failure
  3. CML
  4. Acute leukaemia
A
  1. Acute leukaemia - T - Acute megakaryoblastic leukemia (MKL) occurs in aprox one in 50 to 200 children with Down syndrome. Incidence approx 500x greater in children with than without DS. Acute lymphoblastic leukemia (ALL) is approx 10-20x higher in DS compared with non-DS children. Most children with ALL have anemia and/or thrombocytopenia with either normal or depressed WBC counts and lymphoblasts on peripheral smear (2-3 cells lines down). Most common presenting symptoms of ALL are non specific (fever, bleeding, bone pain, lymphadenopathy).
88
Q
  1. Capillary and cavernous hemangioma – which is false?
  2. Giant hemangioma causes low platelets and bleeding tendency
  3. Cavernous better circumscribed than capillary
  4. Capillary not a recognised entity in liver
A
  1. Capillary not a recognised entity in liver - F - rare but occurs
89
Q
  1. Recent decrease in incidence of Kaposi’s Sarcoma. ?Main reason for this
  2. Decreased incidence EBV
  3. Increased proportion of heterosexuals getting AIDS
  4. New retroviral treatment
  5. ? community education
  6. prophylactic treatment
A
  1. New retroviral treatment - T - highly active anti-retroviral treatment meant to be the reason for reduction of Kaposis
90
Q
  1. Patient thought to have Wegners but CXR normal, most likely evidence to support diagnosis:
  2. CANCA
  3. Nasal ulceration
  4. Antimitochondrial antibodies
A
  1. cANCA - T - +ve in 90%
  2. Patient thought to have Wegners but CXR normal, most likely evidence to support diagnosis: (JS)
  3. cANCA - T - +ve in 90%
  4. Nasal ulceration - F - can occur in other conditions
  5. Antimitochondrial antibodies - F - AMA is seen in 95% of cases of primary biliary cirrhosis not Wegeners
91
Q
  1. Day 5 post AMI, CT for chest pain, least likely:
  2. Pericarditis
  3. Extension of infarct
  4. PE
  5. LV aneurysm
A
  1. LV aneurysm - long term complication.
  2. Day 5 post AMI, CT for chest pain, least likely: (GC)
  3. Pericarditis - T - fibrinous or haemorrhagic, usually dvps within 2-3 days of a transmural infarct and resolves with time.
  4. Extension of infarct - T
  5. PE - T - thromboembolism from mural thrombus in right heart.
  6. LV aneurysm - long term complication.

Other early Cx: contractile dyfxn, arrhythmia, myocardial rupture (ventricular free wall - tamponade; septum - VSD/L-R shunt; papillary m. - severe MR).
Later Cx: ventricular aneurysm, papillary m. dysfunction (MR from ischaemia > rupture), progressive late heart failure.
Risk of Cx and the prognosis after MI dpt on infarct size, site and amt of wall damaged (subendocardial vs. transmural).
[Robbins]

92
Q
  1. Heart on NUCS, 45% reversible defect, 70% ejection fraction, what does it mean for prognosis:???
  2. Decreased EF to 30% if another infarct
  3. Statistically normal finding post MI
  4. Cannot assess prognosis with further MI
A

??

93
Q
  1. PE; diagnosis on CTPA: ???
  2. Infarct
  3. Risk factors
  4. No DVT in legs
A

??

94
Q
  1. Restrictive cardiomyopathy:
  2. RA and myeloma
  3. On some drug
A
  1. RA and myeloma - T - secondary to amyloid which is in both.
  2. Restrictive cardiomyopathy: (JS)
  3. RA and myeloma - T - secondary to amyloid which is in both.
  4. On some drug
    Causes of restrictive cardiomyopathy - idiopathic, radiation fibrosis, amyloidosis, sarcoidosis, metastatic tumour, products of inborn errors of metabolism. Robbins also specifically mentions endomyocardial fibrosis, Loeffler endomyocarditis and endocardial fibroelastosis.
95
Q
  1. Peripheral MCA aneurysm, most likely history:
  2. SLE
  3. PHx of radiation for fibrous dysplasia
  4. HTN
  5. Hx of rheumatic fever or tooth extraction
A
  1. Hx of rheumatic fever or tooth extraction - T - mycotic aneurysms occur in as many as 15% of patients with infective endocarditis, and aneurysms of arteries supplying the brain comprise approx 15% of the aneurysms occurring in IE. The MCA territory is involved 4 times more commonly with mycotic aneurysms than either the anterior or posterior cerebral arteries. In contrast to berry aneurysms, which occur in or near the circle of Willis, mycotic aneurysms occur at secondary branches and bifurcations, usually in the lateral fissure near the trifurcation of the MCA. [eMedicine]
  2. Peripheral MCA aneurysm, most likely history: (GC)
  3. SLE - F - although…. note that the prevalence of SAH due to rupture of intracranial aneurysms is higher in SLE patients than in the general population, thought to be due to focal transmural lupus angiitis. Other factors that may contribute to aneurysm development and rupture include HTN, accelerated atherosclerosis, vasculitis, and concurrent organ dysfunction (eg. CRF). Therefore, SLE patients have 2 concurrent problems related to the cerebral vasculature: (a) abnormal wall mechanics due to inflammatory connective tissue breakdown, and (b) abnormal vascular haemodynamics. [SLE RG 2004]
  4. PHx of radiation for fibrous dysplasia - F - The association between vasculopathy and radiation is well recognized. Generalized arterial disruption and subsequent stenosis and occlusion is most often described. Additional vascular phenomenon, such as pseudoaneurysm formation, aneurysmal dilatation and aneurysmal rupture, are less common. [Acta Neruochirurgica 2005]
  5. HTN - F - Chronic hypertension produces a small vessel vasculopathy characterized by lipohyalinosis, fibrinoid necrosis, and development of Charcot-Bouchard aneurysms, affecting penetrating arteries throughout the brain including lenticulostriates, thalamoperforators, paramedian branches of the basilar a., superior cerebellar aa., and AICAs - intracranial haemorrhage. [eMedicine]
  6. Hx of rheumatic fever or tooth extraction - T - mycotic aneurysms occur in as many as 15% of patients with infective endocarditis, and aneurysms of arteries supplying the brain comprise approx 15% of the aneurysms occurring in IE. The MCA territory is involved 4 times more commonly with mycotic aneurysms than either the anterior or posterior cerebral arteries. In contrast to berry aneurysms, which occur in or near the circle of Willis, mycotic aneurysms occur at secondary branches and bifurcations, usually in the lateral fissure near the trifurcation of the MCA. [eMedicine]
96
Q
  1. Beta thalassemia, which is false:
  2. Splenomegaly
  3. Liver disease
  4. Transfusion disease
  5. Macrocytic anaemia
  6. Major manifests at 6-9mths of age
A
  1. Macrocytic anaemia - F - reduced synthesis of beta-globin leads to inadequate HbA formation, so that the MCHC is low and the cells appear hypochromic and microcytic. Anaemia also occurs due to RBC haemolysis from unbalanced rates of beta and alpha-globin chain synthesis (unpaired alpha-chains form insoluble aggregates that ppt in RBCs and cause membrane damage).
  2. Beta thalassemia, which is false: (GC)
  3. Splenomegaly - T - due to extramedullary haematopoesis.
  4. Liver disease - T - secondary heamochromatosis from transfusions and increased dietary uptake of iron. Also hepatomegaly secondary to EMH.
  5. Transfusion disease - T - if treated with transfusions, survival into 2nd-3rd decade possible, although systemic iron overload can lead to cardiac failure from secondary haemochromotosis. Patients need aggressive treatment with Fe chelators. BMT at an early age is the treatment of choice.
  6. Macrocytic anaemia - F - reduced synthesis of beta-globin leads to inadequate HbA formation, so that the MCHC is low and the cells appear hypochromic and microcytic. Anaemia also occurs due to RBC haemolysis from unbalanced rates of beta and alpha-globin chain synthesis (unpaired alpha-chains form insoluble aggregates that ppt in RBCs and cause membrane damage).
  7. Major manifests at 6-9mths of age - T - HbF synthesis diminishes postnatally. Kids fail to develop normally and have growth retardation from shortly after birth.
    Added options 4 and 5.
97
Q
  1. Mitral valve prolapse with myxomatous degeneration, for excision of abscess (didn’t say where, but assuming peripheral body) which is true:
  2. Not an entity
  3. Risk of bacterial endocarditis
  4. Rheumatic fever is a cause
A
  1. Risk of bacterial endocarditis
    SCS: assuming transient bacteraemia and subsequent colonisation of “at risk” myxoid degenerated valve.
  2. Mitral valve prolapse with myxomatous degeneration, for excision of abscess (didn’t say where) which is true: (JS)
  3. Not an entity
  4. Risk of bacterial endocarditis
  5. Rheumatic fever is a cause - F - myxomatous degeneration of MV is of unknown aetiology but thought to be related to developmental anomaly of connective tissue. Associated with Marfan’s syndrome. Most common form of valvular heart disease in industrial world. Approximately 3% develop complications - infective endocarditis, mitral insufficiency, stroke, arrhythmias.
98
Q
  1. Woman ?young, calcium on aorta on CT, excluding (?smoking) which major modifying risk factors:
  2. HT, obesity, high cholesterol, high HDL, lifestyle, diabetes
  3. Various combinations of these
A
  1. HT, obesity, high cholesterol, high HDL, lifestyle, diabetes

Nonmodifiable risk factors for IHD: age, male gender, family history, genetics
Modifiable risk factors: hyperlipidaemia (low HDL, high LDL), hypertension, smoking, diabetes, obesity, physical inactivity

99
Q
  1. Aortitis 40 yo, clinicians query syphilitic aortitis – which is false?:
  2. Endothelial abnormality on angio
  3. Involve vasa vasorum with infiltrates
  4. Is tertiary manifestation
  5. No because is fibrous so should have no dilation
A
  1. No because is fibrous so should have no dilation - F - endarteritis causes ischaemic injury to the media with inflammation and scarring. This leads to loss of elastic support resulting in aneurysmal dilatation (ascending aorta 60%, arch 30% and descending aorta 10%)
    * *SCS: Radiopaedia. tertiary syphilis “luteic aneurysm”. Classically Involves ascending arch. ass/w AR, aortic vasculitis, coronary ostial stenosis.
    - Additionally, Ascending arch is an atypical, location for intimal calcification- if you see it think Syphilis or Takayasus.
  2. Aortitis 40 yo, clinicians query syphilitic aortitis: (JS)
  3. Endothelial abnormality on angio - T -
  4. Involve vasa vasorum with plasma cell infiltrates - T - particularly involves the vasa vasorum with production of obliterative endarteritis rimmed by an infiltrate of lymphocytes and plasma cells
  5. Is tertiary manifestation - T - is a tertiary manifestation (typically CVS and nervous systems) which occurs 10-30y after infection - so I guess this depends on the age of primary infection… Dahnert gives the age as 40-65y.
  6. No because is fibrous so should have no dilation - F - endarteritis causes ischaemic injury to the media with inflammation and scarring. This leads to loss of elastic support resulting in aneurysmal dilatation (ascending aorta 60%, arch 30% and descending aorta 10%)
100
Q
  1. Vasculitis - PAN, most correct:
  2. Blind
  3. Pulseless
  4. 5 yo.
  5. Involves kidney
A
  1. Involves kidney - T - typically involves kidney and viscera but spares lung. Renal involvement is of small-medium sized muscular arteries (not arterioles, ie. not glomerulonephritis). Hallmark is microaneurysms, typically at branchpoints as a result of segmental transmural inflammation and fibrinoid necrosis. Commonly get arterial occlusions and cortical infarcts.
    [Robbins; Dahnert]
  2. Vasculitis, PAN: (GC)
  3. Blind - T - transient symptoms of cerebral ischemia, including typical spells of transient monocular blindness, are the most commonly presenting neurologic deficit of PAN.
  4. Pulseless - F - small to medium-sized muscular arteries are affected. Takayasu’s involves medium and larger arteries and may manifest as a reduced BP and weaker pulses in the upper limbs relative to LLs.
  5. 5 yo. - F - Robbins: primarily a disease of young adults, but can occur at any age; Dahnert gives a range of 18-81yo. with mean age 55yo.
  6. Involves kidney - T - typically involves kidney and viscera but spares lung. Renal involvement is of small-medium sized muscular arteries (not arterioles, ie. not glomerulonephritis). Hallmark is microaneurysms, typically at branchpoints as a result of segmental transmural inflammation and fibrinoid necrosis. Commonly get arterial occlusions and cortical infarcts.
    [Robbins; Dahnert]
101
Q

Causes of pulmonary hypertension

A

Proceed from right to left heart

Right heart
- left to right shunt

Pulmonary artery

  • atresia/stenosis
  • chronic PE
  • arteritis : Takayasu, PAN, SLE etc

Lung
- any lung disease

Pulmonary vein

  • pulmonary veno occlusive disease
  • venolobar syndrome-atresia, stenosis
  • PAPVR

Left heart

  • left heart failure
  • mitral valve stenosis
  • hypo plastic left heartChest wall
  • kyphoscoliosis
  • Pickwickian syndrome : obesity with hypoventilation syndrome

Others- CRF

102
Q
  1. Regarding polyarteritis nodosa, which is/are false:
  2. The kidneys are most commonly involved at autopsy
  3. Aneursyms are seen at angiography in 50%
  4. 70% are seropositive for Hepatitis B surface antigen
  5. Churg Strauss syndrome frequently involves pulmonary and splenic vessels
  6. Serum antineutrophil antibody titres correlate with disease activity
A
  1. 70% are seropositive for Hepatitis B surface antigen - F - 30% have hepatitis B
  2. Serum antineutrophil antibody titres correlate with disease activity - F - Adelaide path notes: p-ANCA is often present in serum and correlates with disease activity.
    * LW: Robbins states no association with ANCA; hence Gospel.
  3. Regarding polyarteritis nodosa, which is the least correct: (GC)
  4. The kidneys are most commonly involved at autopsy - T - vessels of the kidneys, heart, liver, and GIT are involved in descending order.
  5. Aneursyms are seen at angiography in 50% - T - multiple small intrarenal microaneurysms typically at branchpoints in 50-60% - hallmark feature. Lesions usually involve only part of the vessel circumference; the inflammatory process weakens the arterial wall and can lead to aneurysms or even rupture.
  6. 70% are seropositive for Hepatitis B surface antigen - F - 30% have hepatitis B antigenaemia, and HBsAg-HBsAb immune complexes can be demonstrated in the lesions.
  7. Churg Strauss syndrome frequently involves pulmonary and splenic vessels - T - aka allergic granulomatosis and angiitis; vascular lesions can resemble PAN and microscopic polyangiitis, but in the lung, heart, spleen, peripheral nerves and skin there are also intra- and extravascular granulomas. PAN spares the pulmonary circulation.
  8. Serum antineutrophil antibody titres correlate with disease activity - F - Adelaide path notes: p-ANCA is often present in serum and correlates with disease activity.
    * LW: robbins says false
103
Q
  1. Which of the following is least correct with respects to fatty plaques:
  2. ?% of kids have them
  3. ?Site
A

fatty streaks – not raised, appear in the aortas of some children from 1 but are in all children older than 10 years of age regardless of geography, race, sex or environment. Although fatty streaks may be precursors of plaques, not all are destined. They are often not in areas of later plaque devt.)

104
Q
  1. Regarding cardiomyopathies, which is most correct:
  2. Autosomal recessive cases are seen in >50% of hypertrophic CMP
  3. Heart always increased in weight in dilated CMP
  4. Atria never involved in hypertrophic CMP
  5. Endomyocardial fibrosis is mainly a disease of children and young adults in South America
  6. Dilation of the heart in peripartum CMP is irreversible in the majority
A

**SCS: as per RY🐐. Quoting robbins basic bish 10th ed… “the heart in DCM characteristically is enlarged (up to two or three times the normal weight) and flabby, with dilation of ALL chambers”. Pg 431. Option 4. Whilst seems largely true is felt to the second best answer.

*AJL - I would favour 4 (endomyocardial fibrosis) as being the most correct.
Answer below seems to have confused dilated and hypertrophic CMP.
Dilated CMP has dilated ventricular walls which are normal or thinned.
They have described hypertrophic CMP reasonably well.

Previous answer:

  1. Heart always increased in weight in dilated CMP - T
  2. Atria never involved in hypertrophic CMP - False. LA is dilated due to back pressue and reduced CO/SV.
  3. Endomyocardial fibrosis is mainly a disease of children and young adults in South America - F - technically true but not “most correct”. Primarily a disease of children and young adults in Africa, I [Robbins]; but is also encountered in tropical and subtropical regions elsewhere in the world, including areas in India and South America that are within 15° of the equator [statdx]. EMF is the most common type of restrictive cardiomyopathy worldwide.
  4. Dilation of the heart in peripartum CMP is irreversible in the majority - F - half of these patients will spontaneously recover. Aetiology is multifactorial - preg-assocd HTN, volume overload, nutritional deficiency, metabolic derangement, immunologic response (eg. abnormal cytokine production).
105
Q
  1. Regarding infective endocarditis, which of the following is the least correct:
  2. Right sided heart valves in 50% of narcotic related cases
  3. Diffuse glomerulonephritis in 50% of untreated cases
  4. Positive blood cultures in 85-90% of cases
  5. Strep pneumonia is the leading cause of acute endocarditis
  6. With mechanical prostheses infections are usually located on the margin of the sewing ring
A
  1. Strep pneumonia is the leading cause of acute endocarditis – F – (note that path notes say more than ½ cases due to various streptococci – however most commonly gingival disease) – since 1980s IE due to S aureus (primary pathogen) primary risk factor is intravascular devices (eMed).
  2. Regarding infective endocarditis, which of the following is the least correct: (TW)
  3. Right sided heart valves in 50% of narcotic related cases – T - in 75% of IVDA IE, no underlying valvular abnormalities are noted, and 50% of these infections involve the tricuspid valve (eMed).
  4. Diffuse glomerulonephritis in 50% of untreated cases – T ~30% UpToDate
  5. Positive blood cultures in 85-90% of cases – T – path notes / UpToDate (but varies depending on series and geography)/ eMed
  6. Strep pneumonia is the leading cause of acute endocarditis – F – (note that path notes say more than ½ cases due to various streptococci – however most commonly gingival disease) – since 1980s IE due to S aureus (primary pathogen) primary risk factor is intravascular devices (eMed).
  7. With mechanical prostheses infections are usually located on the margin of the sewing ring – T – endocarditis is located at the prosthesis-tissue intervace, causing a ring intervace (Robbins).
106
Q
  1. Regarding aortic dissection, which is most correct:
  2. 5-10% do not have an obvious intimal tear
  3. Hypertension is an antecedent in 50% of cases
  4. Haemorrhage typically occurs between the inner and middle thirds of the media
  5. Elastic fragmentation of the media is rare at autopsy in patients free of dissection
A
  1. 5-10% do not have an obvious intimal tear - T - There is usually (but not always) an intimal tear that extends into but not through the media of the ascending aorta (Robbins). According to Primer, intimal flap is detectable in 85-90%.
  2. Regarding aortic dissection, which is most correct: (JS)
  3. 5-10% do not have an obvious intimal tear - T - There is usually (but not always) an intimal tear that extends into but not through the media of the ascending aorta (Robbins). According to Primer, intimal flap is detectable in 85-90%.
  4. Hypertension is an antecedent in 50% of cases - F - hypertension is almost always present (>90%), other causes are connective tissue abnormalities such as Marfans
  5. Haemorrhage typically occurs between the inner and middle thirds of the media - F - the dissecting haematoma spreads along the laminar planes of the aorta, between the middle and outer thirds
  6. Elastic fragmentation of the media is rare at autopsy in patients free of dissection - F - cystic medial degeneration (fragmentation and separation of the elastic and fibromuscular elements of the tunica media) is frequently found incidentally at autopsy in patients free of dissection (Robbins
107
Q
  1. Leiomyoma, least common site:
  2. Heart/Myocardium
  3. Blood vessels, vascular smooth muscle
  4. Oesophagus
  5. Stomach
  6. Uterus
A
  1. Heart/Myocardium - T - extremely rare (if at all existent!); there are a case reports of benign metastasizing leiomyomas to heart (most common site is lung); occurring years after fibroid removal.
    Common benign cardiac tumors are myxoma (30% of all primary cardiac tumors), papillary fibroelastoma (8%), rhabdomyoma, fibroma, hemangioma, lipomatous hypertrophy, AV nodal tumors, other (5% or less each). In children, common tumors are rhabdomyoma, fibroma and teratoma.
  2. Leiomyoma, least common site: (GC)
  3. Heart/Myocardium - T - extremely rare (if at all existent!); there are a case reports of benign metastasizing leiomyomas to heart (most common site is lung); occurring years after fibroid removal.
    Common benign cardiac tumors are myxoma (30% of all primary cardiac tumors), papillary fibroelastoma (8%), rhabdomyoma, fibroma, hemangioma, lipomatous hypertrophy, AV nodal tumors, other (5% or less each). In children, common tumors are rhabdomyoma, fibroma and teratoma.
  4. Blood vessels/vascular smooth muscle - F - aka angioleiomyoma, relatively common, painful tumours - usually females 30-60yo., often in soft tissues of lower limbs; tumours in males more common in UL and H&N.
  5. Oesophagus - F - most common benign submucosal tumour of oesophagus; 50% of all benign oesoph tumours. Frequently lower and mid 1/3.
  6. Stomach - F - second most common benign gastric tumour (after polyps); most common of calcified benign tumours; submucosal in 60% (accounts for 90% of submucosal tumours, ulcerated in 50%).
  7. Uterus - F - most common gynae neoplasm; commonest cause for uterine enlargement after pregnancy. Incidence 20-25% of whites, 50% of blacks. Intramural, subserosal, submucosal.
    [Path outlines; Dahnert]
108
Q

Common benign heart tumour

A

Common benign cardiac tumors

  • myxoma (30% of all primary cardiac tumors)
  • papillary fibroelastoma (8%),- rhabdomyoma
  • fibroma
  • hemangioma, lipomatous hypertrophy
  • AV nodal tumors,
  • other (5% or less each).

In children, common tumors are

  • rhabdomyoma
  • fibroma
  • teratoma.
109
Q
  1. Regarding necrosis, which is the most correct:
  2. Liquefactive necrosis is a characteristic of ischaemic destruction of cardiac muscle
  3. Councilman bodies in the liver in toxic or viral hepatitis is an example of apoptosis
  4. The dead cell usually shows decreased eosinophilia
  5. Caseous necrosis is encountered principally in the centre of an Aschoff nodule
  6. Expansion of the nucleus of dead cells with unraveling of the chromatin is called pynkosis
A

b. Councilman bodies occur in the liver in toxic or viral hepatitis is an example of apoptosis. – toxic or immune mediated hepatocyte death leads to formation of shrunken pyknotic and intensely eosinophilic councilman bodies containing fragmented nuclei. Eg in viral hepatitis – apoptotic cells in the liver are known as councilman bodies (Cell injury in viral diseases).

  1. Regarding necrosis, which is the most correct: (TW)
    a. Liquefactive necrosis is a characteristic of ischaemic destruction of cardiac muscle. – F – coagulative necrosis. See liquefactive necrosis in CNS.
    b. Councilman bodies occur in the liver in toxic or viral hepatitis is an example of apoptosis. – toxic or immune mediated hepatocyte death leads to formation of shrunken pyknotic and intensely eosinophilic councilman bodies containing fragmented nuclei. Eg in viral hepatitis – apoptotic cells in the liver are known as councilman bodies (Cell injury in viral diseases).
    c. Dead cells usually show decreased eosinophilia – F – increased eosinophilia
    d. Caesous necrosis is encountered principally in the centre of an aschoff nodule. – F - aschoff body is a focal inflammatory lesion seen in acute rheumatic fever most distinctively in heart with central focus of fibrinoid degeneration surrounded by lymphocytes, occasional plasma cells and anitschkow cells. See caseous necrosis in TB.
    e. Expansion of the nucleus of dead cells with the unraveling of chromatin is called pyknosis – F - pyknosis is irreversible condensation of chromatin in the nucleus of a cell undergoing apoptosis.
    (Robbins 6th ed. Path notes)
110
Q

Pathology Exam – April 2007

  1. Dilated cardiomyopathy and cirrhosis – thing to explain both
    a. Alcohol
A
  1. Dilated cardiomyopathy and cirrhosis – thing to explain both (JS)
  2. Alcohol - T - strongly associated with dilated cardiomyopathy, possibly related to a direct toxic effect of metabolites on myocardium.
111
Q
  1. Takayasu’s, least likely
    a. abnormal pulmonary arteries
    b. narrowing involves abdominal aorta
    b. fibrosing mediastinitis
A

b. fibrosing mediastinitis

Takayasu:

Best diagnostic clue: Wall thickening of large vessels
Thoracic aorta and branches
Pulmonary artery involvement is less common (stat dx)

112
Q
  1. Normal aortic outflow, normal arterial and venous connections , cyanotic
    a. Fallot
    b. Correction of TGA
    c. Tricuspid atresia
    d. PDA
    e. PDV
A

c. Tricuspid atresia – T – 2nd most common cause of pronounced neonatal cyanosis (after transposition). 80% TA without transposition, can have +/-pulmonary stenosis or atresia. Usually small VSD and pulmonary stenosis (75%)

  1. Normal aortic outflow, normal arterial and venous connections , cyanotic (TW)
    a. Fallots Tetralogy – F – most common CHD with cyanosis after 1yr of life. Could argue also incorrect when considering ‘normal arterial and venous connections’ with overriding aorta.
    b. Correction of TGA. – F – corrected – so shouldn’t be cyanotic.
    c. Tricuspid atresia – T – 2nd most common cause of pronounced neonatal cyanosis (after transposition). 80% TA without transposition, can have +/-pulmonary stenosis or atresia. Usually small VSD and pulmonary stenosis (75%)
    d. PDA – F - L to R shunt – not cyanotic.
    e. PDV – F – bypass liver. Asymptomatic to encephalopathy; hypergalactosemia; hypoglycemia; and, rarely, hypoxemia (AJR 2005).
113
Q
  1. Aortitis 40 yo female, clinicians query syphilitic aortitis (which is least correct?):
    a. Endothelial abnormality on angio
    b. Involve vasa vasorum with plasma cell infiltrates
    c. Is tertiary manifestation
    d. No because is fibrous so should have no dilation
A
  1. No because is fibrous so should have no dilation - F - endarteritis causes ischaemic
  2. Aortitis 40 yo female, clinicians query syphilitic aortitis (which is correct?): (JS)
  3. Endothelial abnormality on angio - T -
  4. Involve vasa vasorum with plasma cell infiltrates - T - particularly involves the vasa vasorum with production of obliterative endarteritis rimmed by an infiltrate of lymphocytes and plasma cells
  5. Is tertiary manifestation – not likely too young - T - is a tertiary manifestation (typically CVS and nervous systems) which occurs 10-30y after infection - so I guess this depends on the age of primary infection… Dahnert gives the age as 40-65y.
  6. No because is fibrous so should have no dilation - F - endarteritis causes ischaemic injury to the media with inflammation and scarring. This leads to loss of elastic support resulting in aneurysmal dilatation (ascending aorta 60%, arch 30% and descending aorta 10%)
114
Q
  1. A patient has prominent pulmonary artery, which of the following histories is least likely cause of pulmonary arterial hypertension? (asked as a positive question and a negative question)
    a. 38 yo smoker with basal emphysema
    b. 32 yo female whose mother died of primary pulmonary arterial hypertension
    c. 2 yo refugee with pansystolic machinery type cardiac murmur
    d. 30 yo female diabetic with chronic renal failure
    e. 22 yo with SLE with recurrent right leg swelling
A

*AJL
Least likely - Diabetic with CRF.
Most likely - Maybe SLE patient.
Cant be 2yo refugee as Eisenmenger is rare before age 2.

  • WJI disagree with logic. Eisenmengers is RV pressure>LV pressure and reversal of shunt. Pulmonary HTN is PA pressure >25mmHg. This occurs before Eisenmengers and I would think is pretty common with a significant L to R shunt; probably even the most common of all below options.
    4. 30 yo female diabetic with chronic renal failure
  1. A patient has prominent pulmonary artery, which of the following histories is least likely cause of pulmonary arterial hypertension? (asked as a positive question and a negative question) (JS)
  2. 38 yo smoker with basal emphysema - T - alpha 1 AT leading to increased pulmonary artery pressure
  3. 32 yo female whose mother died of primary pulmonary arterial hypertension – T - 15-20% have a familial form which is only recently characterised (emedicine).
  4. 2 yo refugee with pansystolic machinery type cardiac murmur - T - PDA leading to increased pulmonary venous pressure (*AJL - Eisenmenger is uncommon before age 2 (RP))
  5. 30 yo female diabetic with chronic renal failure - ??
  6. 22 yo with SLE with recurrent right leg swelling - T - recurrent pulmonary emboli due to lupus anticoagulant

Opinions please - 2 was previous thought to be false but PPH can be familial…. What do you guys think??
I agree with option 4 as least likely. GC.

115
Q
  1. Mass seen in lower lobe of lung in patient with AIDS; which is true?
    a. Increased proportion of heterosexuals getting AIDS
    b. Kaposi sarcoma is less common if patient is on retroviral treatment HAART
    c. prophylactic treatment
    d. Mass resulting from PCP infection.
A

d. Mass resulting from PCP – F - most common radiographic abnormality are diffuse, bilateral interstitial or alveolar infiltrates. Other less common presentation include PTX, lobar or segmental infiltrates, cysts, nodules (UpToDate)

  1. Mass seen in lower lobe of lung in patient with AIDS; which is true? Not sure of the question (TW)
    a. Increased proportion of heterosexuals getting HIV – T - …In summary, heterosexual transmission of HIV and AIDS appears to be increasing in the USA and may be grossly underestimated by our present surveillance system. Postgraduate Medical Journal 2003
    b. Kaposi sarcoma is less common if patient on retroviral treatment HAART – T – among HIV infected patients, HAART is assoc with decreased proportion of new KS cases, regression in size of existing KS lesions, and possible improved survival in pts with KS with or without chemoTx (UpToDate).
    c. Prophylactic treatment.
    d. Mass resulting from PCP – F - most common radiographic abnormality are diffuse, bilateral interstitial or alveolar infiltrates. Other less common presentation include PTX, lobar or segmental infiltrates, cysts, nodules (UpToDate)
    Pulmonary KS in AIDs patients – typically interstitial or nodular pattern. 60% have patchy interstitial infiltrates in a perihilar distribution on CXR.
    Should this be “which is false”? And perhaps in option c they’re referring to prophylactic aerosolized pentamidine for PCP (UL distribution)? A stem could be added regarding AIDS-related lymphoma (which may manifest as solitary/multiple well-defined nodules with rapid doubling time; or alveolar infiltrates; or paraspinal mass - these could account for a LL mass).
116
Q
  1. Regarding pulmonary emboli and infarcts, which of the following is MOST CORRECT:
    a. Majority (>50%) result in white infarcts because it is an end circulation
    b. 75% are in the lower lobes
    c. Pulmonary haemorrhage suggest infarction
    d. The presence of viable alveoli pneumocytes/endothelium in an area of haemorrhage is against a diagnosis of PE
    e. >50% of infarcts are solitary
A
  1. 75% are in the lower lobes -T 7.
  2. Regarding pulmonary emboli and infarcts, which of the following is MOST CORRECT: (JS)
  3. Majority (>50%) result in white infarcts because it is an end circulation - F - Classically haemorrhagic
  4. 75% are in the lower lobes -T
  5. Pulmonary haemorrhage suggest infarction - F - the bronchial arteries can sustain the lung parenchyma despite obstruction of pulmonary arterial system and under these circumstances, haemorrhages may occur with no infarction. Only about 10% of emboli cause infarction (typically when circulation is inadequate such as heart or lung disease). (Robbins)
  6. The presence of viable alveoli pneumocytes/endothelium in an area of haemorrhage is against a diagnosis of PE - F - haemorrhages (I presume related to pe) are distinguished by preservation of pulmonary architecture
  7. > 50% of infarcts are solitary - F - in more than half of cases, multiple lesions occur
117
Q
  1. Cavernous Angioma – What is not typical
    a. detectable at angiography
    b. Bleeding tendency
    c. No intervening brain
    d. Pseudo capsule + surrounding hemosiderin laden macrophages
    e. Associated venous angioma
A
  1. detectable at angiography F - angiographically occult lesions
  2. Cavernous Angioma – What is not typical (JS)
  3. detectable at angiography F - angiographically occult lesions
  4. Bleeding tendency - T - contain blood degredation products of varying ages, haemorrhage risk is 0.5-1% per year but occult bleeds are more common (Osborne)
  5. No intervening brain - T - dilated endothelial cell-lined spaces with no normal brain within the lesion
  6. Pseudo capsule + surrounding hemosiderin laden macrophages - T - A low signal haemosiderin ring is typical on MR
  7. Associated venous angioma - T - up to 1 third of DVAs are associated with cavernous angiomas (Osborne)
118
Q
  1. Female has an irregularity of the aortic arch for which the following: past dissection, Takayasus arteritis , and giant cell arteritis are considered .Which of the following statements is most correct?
    a. If patient is < 40 yrs old , past dissection should be discounted.
    b. If the patient is female , Takayasu arteritis would be less likely.
    c. If the patient is over 70 yrs Takayasu arteritis is strongly favoured .
    d. If there is renal vessel narrowing, past dissection would be strongly favoured.
    e. If over 50yrs and giant cells on biopsy then giant cell arteritis should be considered.
A

e. If over 50yrs and giant cells on biopsy then giant cell arteritis should be considered. – T - affects older people (rare <50yo). Giant cells present in only 2/3rds. Most common systemic vasculitis in adults.

  1. Female has an irregularity of the aortic arch for which the following: past dissection, Takayasus arteritis , and giant cell arteritis are considered .Which of the following statements is most correct? (TW)
    a. If patient is < 40 yrs old, past dissection should be discounted – F – although most are 40-60yo men with HT, can have connective tissue disorders (eg Marfans), or post arterial cannulation, during pregnancy (path notes)
    b. If the patient is female , Takayasu arteritis would be less likely – F – would be more likely. TA affects females 15-40yo, most common in Asia.
    c. If the patient is over 70 yrs Takayasu arteritis is strongly favoured - F – less favoured. See ans to (a)
    d. If there is renal vessel narrowing, past dissection would be strongly favoured. – F - Takayasu’s arteritis classically involves arch and ostis of major branchs, however 1/3 also affect the remainder of the aorta and its branches. Some cases are limited to descending thoracic and abdominal aorta.
    e. If over 50yrs and giant cells on biopsy then giant cell arteritis should be considered. – T - affects older people (rare <50yo). Giant cells present in only 2/3rds. Most common systemic vasculitis in adults.
119
Q
  1. Concerning PE which is correct?
    a. 40% lead to infarction.
    b. Emboli affect upper, mid and lower lobe zones equally .
    c. In the young PE more frequently leads to infarction.
    d. In the prescence of a predisposing cause , secondary episodes may occur in up to 30%.
    e. Pulmonary haemorrhage implies reperfusion of an infarct.
A

d. In the presence of a predisposing cause, secondary episodes may occur in up to 30% - T – 30% develop a second embolus (path notes).

  1. Concerning PE which is correct? (TW)
    a. 40% lead to infarction – F – dual blood supply. Embolism without infarction 90%, embolism with infarction 10-60% (D 6th ed.). Path notes say 10% only.
    b. Emboli affect upper, mid and lower lobe zones equally – F – bilat 45%, right lung only 36%, left lung only 18%. (LL > UL > ML) LLL 26%, RLL 25%, RUL 16%, LUL 14%, RML 9%.
    c. In the young PE more frequently leads to infarction – F - see increased pulmonary infarction if abnormal bronchial supply, usually assoc with underlying heart – lung disease. Would be less in younger groups.
    d. In the presence of a predisposing cause, secondary episodes may occur in up to 30% - T – 30% develop a second embolus (path notes).
    e. Pulmonary haemorrhage implies reperfusion of an infarct. – F – pulmonary haemorrhage is much commoner than infarction (path notes). In patients with adequate CV function, the bronchial artery supply can often sustain the lung parenchyma despite obstruction to the pulmonary arterial system Under these circumstances, haemorrhages may occur, but there is no infarction (Robbins)
120
Q
  1. Mass in lower lobe of lung in patient with AIDS , which is false?
    a. Increased proportion of heterosexuals getting HIV.
    b. Kaposis sarcoma is less common if patient on retroviral treatment HAART.
    c. Prophylactic treatment with pentaminidine.
    d. AIDS related lymphoma.
A

c. Prophylactic treatment with pentamidine - F - redistribution of infection to upper lobes.

  1. Mass in lower lobe of lung in patient with AIDS, which is false? (TW)
    a. Increased proportion of heterosexuals getting HIV – T - …In summary, heterosexual transmission of HIV and AIDS appears to be increasing in the USA and may be grossly underestimated by our present surveillance system. Postgraduate Medical Journal 2003
    b. Kaposi sarcoma is less common if patient on retroviral treatment HAART – T – among HIV infected patients, HAART is assoc with decreased proportion of new KS cases, regression in size of existing KS lesions, and possible improved survival in pts with KS with or without chemoTx (UpToDate).
    c. Prophylactic treatment with pentamidine - F - redistribution of infection to upper lobes.
    d. AIDS related lymphoma – T - Thoracic involvement is present in 20-40% of patients with ARL. Primary pulmonary ARL accounts for only 10-15% of cases. Multiple pulmonary nodules and masses, rangin in size from 1-5cm, are seen most commonly on radiographs or CT (Thoracic Imaging).
    PCP: most common radiographic abnormality are diffuse, bilateral interstitial or alveolar infiltrates. Other less common presentation include PTX, lobar or segmental infiltrates, cysts, nodules (UpToDate). Atypical pattern (5% as per Dahnert) of isolated lobar disease / focal parenchymal opacities.
    Pulmonary KS in AIDs patients – typically interstitial or nodular pattern. 60% have patchy interstitial infiltrates in a perihilar distribution on CXR.
121
Q
  1. In an immunocompetent patient, which of the following is least likely to cause a pericardial effusion? (TW)
    a. SLE
    b. Myxoedema.
    c. Post MI.
    d. Coxsackie virus.
    e. Cryptococcus infection.
A

e. Cryptococcus infection.

Pericardial effusion in hypothyroidism is common and recognized consequence of myxedema.
Dahnert 6th: Dressler’s, myxedema, AMI, infection (viral, pyogenic, tuberculous), collagen vascular disease SLE.
Multiple reports of cryptococcus pericardial effusion – however often assoc with immunosuppresion / HIV.

122
Q
  1. Renal vein thrombosis is least likely in which of the following settings? (TW)
    a. Membranous glomerulonephritis.
    b. Amyloid
    c. Mitral stenosis.
    d. Oral contraceptive pill.
    e. Non proliferative glomerulonephritis.
A

c. Mitral stenosis.

Dahnert 6th / eMed – membranous + membranoproliferative GN. Nephrotc symdrome (usually caused by non-proliferative GN = e.g. minimal change, FSGS, membranous GN). Lupoid nephrosis. Amyloidosis. Hypercoaguable states – OCP.